You are on page 1of 80

CHAPTER 1

INTRODUCTION TO ENGINEERING ECONOMICS

Fundamentally, engineers are designers, builders, and creators. Because it costs


money to design, develop, manufacture and sell goods (and/or services) that
engineers produce, the only way to be effective at doing this whole cycle is to
understand economics. Having sound understanding of how economy works is critical
to the success of the engineer’s undertakings.

This chapter discusses the elements of economics and the interaction between its
various components. This is followed by an analysis of the need of engineering
economics. Later, elements of cost and break-even analysis are presented.

After completing this chapter, the student shall be able to:

 Define economics and engineering economics;


 Differentiate goods from services;
 Differentiate necessities from luxuries;
 Identify the type of market structure;
 State the law of supply and demand and law of diminishing return;
 Define cost and its elements;
 Perform calculations on problems requiring break-even analysis; and
 Understand the types of engineering economic decisions.

Lesson 1: Economics

Economics is the science that deals with the production and consumption of goods
and services and the distribution and rendering of these for human welfare.

Figure 1. Flow of goods, services, resources and money payments in a simple


economy

1
The flow of goods, services, resources and money payments in a simple economy are
shown in Figure 1. Households and businesses are the two major entities in a simple
economy. Business organizations use various economic resources like land, labor and
capital which are provided by households to produce consumer goods and services
which will be used by them. Business organizations make payment of money to the
households for receiving various resources. The households in turn make payment of
money to business organizations for receiving consumer goods and services. This
cycle shows the interdependence between the two major entities in a simple economy.

Lesson 2: Goods vs. Services

Goods are basically objects or products which have to be manufactured, stored,


transported, marketed and sold. Coca-cola, Toyota, Nike are some companies
manufacturing goods.

Commodities are economic goods that are interchangeable with other goods of the
same type. Most commodities are raw materials, basic resources, agricultural, or
mining products such as iron ore, sugar, and rice. Commodities can also be mass
produced unspecialized products such as chemicals and computer memory.

Services on the other hand are output of individuals and they can be a collective or
individualistic action or performance by an individual. For example, a barber or a
chartered accountant are giving individual services. Airlines on the other hand have
airplanes which is a product but travelling by airplanes is a service.

Lesson 3: Necessities vs. Luxuries

Necessities are those products or services that are required to support human life and
activities, that will be purchased in somewhat the same quantity even though the price
varies considerably.

Examples: food, clothes, water, shelter

Luxuries are those products or services that are desired by humans and will be
purchased if money is available after the required necessities have been obtained.

Examples: fine jewelry, overseas trips, designer clothes, luxury car

Lesson 4: Demand

Demand is the quantity of a certain commodity that is bought at a certain price at a


given place and time.

An elastic demand is one in which a slight change in the price will lead to drastic
change in the demand for the product. It differs from an inelastic demand in the sense
that a change in price may have no or little effect on the demand of consumers.

Demand curve is flatter when the demand is elastic. Conversely, if the demand is
inelastic, the slope will be steep as can be seen in the figure below.

2
Figure 2. Elastic vs. Inelastic Demand

Luxury goods, goods with many close substitutes have an elastic demand whereas
necessities, goods with no substitutes have an inelastic demand. For example - Coke,
television, airline tickets etc. have an elastic demand whereas commodities like food
grains, salt, tobacco, medicine have an inelastic demand.

Lesson 5: Law of Supply and Demand

An interesting aspect of the economy is that the demand and supply of a product are
interdependent and they are sensitive with respect to the price of that product.

Figure 3. Demand and Supply Curve

The law of supply states that the quantity of a good supplied (i.e., the amount
producers offer for sale) rises as the market price rises, and falls as the price falls.
Conversely, the law of demand says that the quantity of a good demanded falls as the
price rises, and vice versa.

The point of intersection of the supply curve and the demand curve is known as the
equilibrium point. At the price corresponding to this point, the quantity of supply is equal
to the quantity of demand.

Factors influencing demand:

3
The shape of the demand curve is influenced by the following factors:
 Income of the people
 Prices of related goods
 Tastes of consumers

Factors influencing supply:

The shape of the supply curve is affected by the following factors:


 Cost of the inputs
 Technology
 Weather
 Prices of related goods

Lesson 6: The Law of Diminishing Returns

“When the use of one factors of production is limited, either in increasing cost or by
absolute quantity, a point will be reached beyond which an increase in the variable
factors will result in a less than proportionate increase in output.”

Production requires two things, labor and machines.

Now imagine this situation. You have 4 packaging machines in a candy shop. Every
time the salesmen packs something, he uses the packaging machine. There is one
salesman, and four machines. The machines are being underutilized, hence more
salesmen can be hired. Assume now there are 10 salesmen, where each person packs
and hardly has to wait for the other one to finish. Thus total units sold increases and
productivity rises. Now assume there are 40 salesmen. In order to use one machine,
there is a long queue of the salesmen to use it. Thus indicating a waste of their time
and lower productivity for additional men hired. So eventually, although we do hire
more labor, the marginal productively of labor falls as there are more number of
salesmen per machine.

Thus, diminishing returns states that with fixed capital, as we hire more and more
inputs, productivity first rises and then eventually begins to fall.

Lesson 7: Market

A market is a place where two parties can gather to facilitate the exchange of goods
and services. The parties involved are usually buyers and sellers. The market may be
physical like a retail outlet, where people meet face-to-face, or virtual like an online
market, where there is no direct physical contact between buyers and sellers.

Market Structures:

 Perfect competition is a market structure in which there is a large number of


buyers and sellers and there is nothing to prevent the entry of additional sellers
in the market. Examples: online shopping, agricultural markets

 Monopoly is a market structure with only one seller of a product and where
there are no close substitutes. A monopoly may also prevent the entry of all
other sellers into the market. Examples: Microsoft, Google

4
 Oligopoly is a market structure with only a few sellers, each offering a product
similar or identical to the others. Examples: Oil and airline industries. If there
are only two sellers dominating or having exclusive control over a market it is
called duopoly. Examples: Pepsi and Coca Cola, Android and Apple, Globe
and PLDT

Lesson 8: Engineering Economics

Engineering economics is defined as the analysis and the evaluation of the factors that
will affect the economic success of engineering projects to the end that a
recommendation can be made which will ensure the best use of capital.

Why is the study of engineering economics important?

 Engineers are called upon to analyze and select the most economical
alternative among several design alternatives.
 Engineers often play a major role in investment decisions based on the analysis
and design of new product or processes.
 Decisions made by engineer during the engineering phase of a product’s
development determine the majority of the costs of manufacturing the product.

Lesson 9: Cost

Cost is the monetary value of goods and services that producers and consumers
purchase. More conventionally, cost has to do with the relationship between the value
of production inputs and the level of output. Total cost refers to the total expense
incurred in reaching a particular level of output.

Classification:
1. Variable cost – varies with volume of production
 Direct material cost – are those materials that are used to produce the product
 Direct labor cost – is the amount of wages paid to the direct labor involved in
the production activities
 Direct expenses – are those expenses that vary in relation to the production
volume, other than direct material costs and labor costs
2. Overhead cost – is the fixed cost irrespective of the production of volume
 Factory overhead – costs incurred during the manufacturing process, not
including the costs of direct labor and direct materials. This includes rent,
utilities, building insurance, equipment maintenance and depreciation, and
insurance on production facilities and equipment.
 Administration overhead – all the costs that are incurred in administering the
business
 Selling overhead – is the total expense that are incurred in the promotional
activities and the expenses relating to the sales force
 Distribution overhead – is the total cost of shipping from the factory site to the
customer site

Other Costs/Revenues

Marginal Cost. Marginal cost of a product is the cost of producing an additional unit of
that product. Let the cost of producing 20 units of a product be ₱10,000, and the cost

5
of producing 21 units of the same product be ₱10,045. Then the marginal cost of
producing the 21st unit is ₱45.

Marginal Revenue. Marginal revenue of a product is the incremental revenue of selling


an additional unit of that product. Let, the revenue of selling 20 units of a product be
₱15,000 and the revenue of selling 21 units of the same product be ₱15,085. Then,
the marginal revenue of selling the 21st unit is ₱85.

Sunk Cost. This is known as the past cost of an equipment/asset. Let us assume that
an equipment has been purchased for ₱1,00,000 about three years back. If it is
considered for replacement, then its present value is not ₱1,00,000. Instead, its
present market value should be taken as the present value of the equipment for further
analysis. So, the purchase value of the equipment in the past is known as its sunk cost.

Opportunity Cost. Opportunity cost of an alternative is the return that will be foregone
by not investing the same money in another alternative. Consider that a person has
invested a sum of ₱50,000 in shares. Let the expected annual return by this alternative
be ₱7,500. If the same amount is invested in a fixed deposit, a bank will pay a return
of 18%. Then, the corresponding total return per year for the investment in the bank is
₱9,000. This return is greater than the return from shares. The foregone excess return
of ₱1,500 by way of not investing in the bank is the opportunity cost of investing in
shares.

Lesson 10: Break-Even Analysis

The main objective of break-even analysis is to find the cut-off production volume from
where a firm will make profit. Let

s – selling price per unit


v – variable cost per unit
FC – fixed cost per period
Q – volume of production

The total sales or revenue (S) of the firm is given by the following formula:

𝑇𝑜𝑡𝑎𝑙 𝑠𝑎𝑙𝑒𝑠 = 𝑠 × 𝑄

The total cost of the firm for a given production volume is given as

𝑇𝑜𝑡𝑎𝑙 𝑐𝑜𝑠𝑡 = 𝑇𝑜𝑡𝑎𝑙 𝑓𝑖𝑥𝑒𝑑 𝑐𝑜𝑠𝑡 + 𝑡𝑜𝑡𝑎𝑙 𝑣𝑎𝑟𝑖𝑎𝑏𝑙𝑒 𝑐𝑜𝑠𝑡

𝑇𝑜𝑡𝑎𝑙 𝑐𝑜𝑠𝑡 = 𝐹𝐶 + 𝑣 × 𝑄

The linear plots of the above two equations are shown below. The intersection point of
the total sales revenue line and the total cost line is called the break-even point. The
corresponding volume of production on the X-axis is known as the break-even sales
quantity. At the intersection point, the total cost is equal to the total revenue. This
point is also called the no-loss or no-gain situation.

6
Figure 4. Break-even Chart

For any production quantity which is less than the break-even quantity, the total cost
is more than the total revenue. Hence, the firm will be making loss. For any production
quantity which is more than the break-even quantity, the total revenue will be more
than the total cost. Hence, the firm will be making profit.

𝑃𝑟𝑜𝑓𝑖𝑡 = 𝑆𝑎𝑙𝑒𝑠 − (𝐹𝑖𝑥𝑒𝑑 𝑐𝑜𝑠𝑡 + 𝑉𝑎𝑟𝑖𝑎𝑏𝑙𝑒 𝑐𝑜𝑠𝑡)

Examples:

1. The cost of producing a small transistor radio set consists of ₱23.00 for labor and
₱37.00 for materials. The fixed charges in operating the plant are ₱100,000 per month.
The variable cost is ₱1.00 per set. The radio set can be sold for ₱75.00 each.
Determine how many sets must be produced per month to break-even.

Solution:
Let Q – be the number of sets to be produced per month to break-even

𝑇𝑜𝑡𝑎𝑙 𝑠𝑎𝑙𝑒𝑠 = ₱75 (Q)

𝑇𝑜𝑡𝑎𝑙 𝑐𝑜𝑠𝑡 = ₱100,000 + (₱23 + ₱37 + ₱1)(Q) = ₱100,000 + ₱61 (Q)

𝑇𝑜 𝑏𝑟𝑒𝑎𝑘 − 𝑒𝑣𝑒𝑛:

𝑇𝑜𝑡𝑎𝑙 𝑠𝑎𝑙𝑒𝑠 = 𝑇𝑜𝑡𝑎𝑙 𝑐𝑜𝑠𝑡


₱75 (Q) = ₱100,000 + ₱61 (Q)
₱14 (Q) = ₱100,000
Q = 7,143 sets

Lesson 11: Engineering Economic Decisions

The term "engineering economic decision" refers to all investment decisions relating
to engineering projects. The most interesting facet of an economic decision, from an

7
engineer's point of view, is the evaluation of costs and benefits associated with making
a capital investment.

The five main types of engineering economic decisions are:

1. Service or Quality Improvement. Investments in this category include any


activities to support the improvement of productivity, quality, and customer
satisfaction in the service sector, such as in the financial, healthcare, and retail
industries.

2. New Products or Product Expansion. Investments in this category are those


that increase the revenues of a company if output is increased. There are two
common types of expansion decision problems. The first type includes
decisions about expenditures to increase the output of existing production or
distribution facilities. The second type of decision problem includes the
consideration of expenditures necessary to produce a new product or to
expand into a new geographic area.

3. Equipment and Process Selection. This class of engineering decision


problem involves selecting the best course of action when there are several
ways to meet a project's requirements. Which of several proposed items of
equipment shall we purchase for a given purpose? The choice often hinges on
which item is expected to generate the largest savings (or return on the
investment).

4. Cost Reduction. A cost-reduction project is a project that attempts to lower a


firm's operating costs. Typically, we need to consider whether a company
should buy equipment to perform an operation now done manually or spend
money now in order to save more money later.

5. Equipment Replacement. This category of investment decisions involves


considering the expenditure necessary to replace worn-out or obsolete
equipment.

A Sample Engineering Economic Decision:

A local manufacturing firm produces crankshafts. They have been using a lathe (a
type of machine performing various operations) that was purchased twelve years ago.
As the production engineer in charge of producing the crankshafts, you expect demand
to continue into the foreseeable future. Over the past two years the lathe has broken
frequently and has now stopped operating altogether. You must now decide to repair
the lathe or purchase a new lathe or if a more efficient lathe will be available in the
future you may wait to buy the new lathe in a couple of years. The economic decision
is whether you should make the considerable investment in a new lathe now or later.
Complicating the decision is the fact that the demand for crankshafts has begun to
decline.

What do we need to know to make a decision?

We basically have two alternatives:

1. Repair the existing lathe


2. Purchase a new lathe now or later

8
For the existing lathe we need to know:

 The cost of repairing the lathe.


 Frequency or probability of breakdown of the lathe.
 Time when the lathe becomes obsolete.
 Estimate the future demand for the crankshafts.
 Estimate the salvage value or cost to remove the old lathe.

For the new lathe we need to know:

 The cost of the new lathe including installation.


 The cost of educating the operator to use the new lathe.
 How often will the lathe require repairs and how much will cost?
 The estimated economic and service life of the new lathe.
 The estimated salvage value of the new lathe.
 Are there any additional operational costs to the new lathe over the old lathe?
 Will the operational costs increase as the new lathe ages?
 If so how much will they increase over the years?
 Will the new lathe produce more crankshafts to increase income?
 What are the tax implications of purchasing the new lathe?
 Will the new lathe enhance or lower employee morale?
 What are the financing costs of purchasing the new lathe?

Economic Decision-Making Process

1. Collect relevant information regarding the project:

 Initial Costs: design, manufacturing, marketing, testing, installation,


construction, taxes, down payments, etc.
 Annual Costs: operating, maintenance, finance payments, insurance, income
taxes, etc.
 Periodic Costs: overhauls, improvements and modifications.
 Annual Receipts: income generated and savings due to increased productivity.
 Salvage Value: income generated by sale or cost to remove obsolete
equipment.
 Financing Method and Interest Rate.

2. Recognize and Define Feasible Alternatives:

 Consider all possible options including the “do nothing” alternative.


 The generated alternatives may not be economically viable.
 Examine each alternative and remove any overlapping options.
 If the productivity is the about the same for each alternative, focus only on the
costs.

3. Consider the future consequences of each alternative:

 Look at environmental impacts, effects on employee productivity, marketing


potential, public relations, etc.

4. Determine whose viewpoint is to be selected when evaluating alternatives:

9
 Private vs. Governmental viewpoint. Very important when the public sector is
involved.

5. The consequences of each alternative must be expressed in monetary units for us


“pesos”:

 You must consider the “time value of money”. It is sometimes very difficult to
put a monetary value on a consequence.

6. When comparing alternatives, focus on the differences between the alternatives:

 The past is common to all alternatives: look towards the future when comparing
alternatives. There can be no consequences before the moment of decision.

7. Develop several criteria to be used in evaluating the alternatives:

 Primary criterion: Economic analysis of alternatives based on a Minimum


Attractive Rate of Return (MARR) value.
 Secondary criterion: Look at “intangibles” and “side-effects”.

8. Evaluate each alternative, using a sensitivity analysis to enhance the evaluation:

 Evaluation methods include: Present Worth (PW), Annual Worth (AW), Future
Worth (FW), Rate of Return (ROR), Capitalized Cost (CC), Benefit/Cost Ratio
(B/C) and Payback Period Analysis using a Minimum Attractive Rate of Return
(MARR).

9. Select the best alternative based on the economic analysis while remembering the
secondary criterion.

10
CHAPTER EXERCISES

Objective Test 1

1. What refers to the goods and services that are required to support human life, needs,
and activities?

a. producer products b. consumer products


c. luxury d. necessity

2. What is considered as the basic consuming or demanding unit of a commodity?

a. seller b. manufacturer c. producer d. buyer or consumer

3. What refers to the goods and services that are desired by human and will be
acquired only after all the needs have been satisfied?

a. producer products b. consumer products


c. luxury d. necessity

4. What refers to the exchange mechanism that brings together the sellers and buyers
of a product, factor of production, or financial security?

a. mall b. market c. store d. office

5. What market situation exist when there are many buyers and many sellers?

a. perfect competition b. oligopoly c. oligopsony d. monopoly

6. Oligopoly exists when there is/are:

a. few sellers and few buyers b. few sellers and many buyers
c. many sellers and few buyers d. one seller and few buyers

7. Duopoly is a market situation where there is/are:

a. few seller and two buyers b. two sellers and many buyers
c. many sellers and two buyers d. one seller and two buyers

8. If there only one seller and many buyers, the market situation is

a. duopsony b. oligopoly c. duopoly d. monopoly

9. “When one of the other factors of production is fixed in quantity or is difficult to


increase, increasing the other factors of production will result in a less than
proportionate increase in output”. This statement is known as the:

a. Law of supply b. Law of demand


c. Law of diminishing return d. Law of supply and demand

10. What refers to the need, want, or desire for a product backed by the money to
purchase it?

11
a. supply b. demand c. product d. good

11. What is the amount of a product available for sale?

a. supply b. demand c. product d. good

12. The following factors influence supply EXCEPT:

a. technology b. cost of raw materials


c. taste of consumers d. weather

13. What refers to an imaginary cost representing what will not be received if a
particular strategy is rejected?

a. opportunity cost b. null cost c. marginal cost d. sunk cost

14. All are classified under direct labor expenses EXCEPT one. Which one?

a. inspection b. testing b. assembly d. supervision

15. Which of the following products may result in an elastic demand?

a. food b. medicine c. softdrinks d. water

16. What is defined as the analysis and evaluation of the monetary consequences by
using the theories and principles of economics to engineering applications, designs,
and projects?

a. economic analysis b. engineering cost analysis


c. engineering economics d. design cost analysis

Problem Solving 1

1. A small shop in Bulacan fabricates portable threshers for palay producers in the
locality. The shop can produce each thresher at a labor cost of ₱1,800. The cost of
materials for each unit is ₱2,500. The variable costs amount to ₱650 per unit, while
fixed charges incurred per annum totals ₱69,000. If the portable threshers are sold at
₱7,800 per unit, how many units must be produced and sold per annum to break-even?
Ans. 25 units

2. A leading shoe manufacturer produces a pair of Lebron James signature shoes at


a labor cost of ₱900 a pair and a material cost of ₱800 a pair. The fixed charges on
the business are ₱5,000,000 a month and the variable costs are ₱400 a pair. Royalty
to Lebron James is ₱1,000 per pair of shoes sold. If the shoes sell at ₱5,000 a pair,
how many pairs must be produced each month for the manufacturer to break-even?
Ans. 2,632

12
CHAPTER 2

INTEREST FORMULAS AND THEIR APPLICATIONS

In engineering economic analysis, taking into account the effect of interest operating
on sums of cash over time is so important. Interest formulas allow us to place different
cash flows received at different times in the same time frame and to compare them.
As will become apparent, almost our entire study of engineering economics is built on
the principles introduced in this chapter.

It is also good to note that the concepts presented in this chapter are equally applicable
to engineering, business, and financial system problems.

After completing this chapter, the student shall be able to:

 Solve simple and compound interest problems involving practical applications;


 Construct a cash flow diagram taking into account the receipts and
disbursements;
 Understand the time value of money and the concept of equivalence;
 Perform calculations in finding present worth, future worth, annuities, and
gradient series;
 Solve problems on capitalized cost; and
 Create an amortization schedule.

Lesson 1: Interest

Interest is a fee that is charged for the use of someone else's money. The size of the
fee will depend upon the total amount of money borrowed and the length of time over
which it is borrowed.

Whenever money is borrowed or invested, one party acts as the lender and another
party as the borrower. The lender is the owner of the money, and the borrower pays
interest to the lender for the use of the lender's money. Most problems here in
engineering economics is either you borrow money from a bank or you invest your
money in a bank. In the first case, you are the borrower and the bank is the lender.
Meanwhile in the second one, you are the lender and the bank is the borrower.

Interest rate

If a given amount of money is borrowed for a specified period of time (typically, one
year), a certain percentage of the money is charged as interest. This percentage is
called the interest rate.

Lesson 2: Simple Interest

Simple interest is defined as a fixed percentage of the principal (the amount of money
borrowed), multiplied by the life of the loan. Thus,

𝐼 = 𝑃𝑖𝑛
The accumulated amount will be

13
𝐹 =𝑃+𝐼
𝐹 = 𝑃 + 𝑃𝑖𝑛
𝐹 = 𝑃(1 + 𝑖𝑛)

where:
I – interest
P – principal or present worth
n – number of interest periods
i – rate of interest per interest period
F – accumulated amount or future worth

Types of Simple Interest:

1. Ordinary simple interest – is computed on the basis of 12 months of 30 days each


or 360 days a year (also known as the banker’s year).
1 interest period = 360 days

2. Exact simple interest – is based on the exact number of days in a year, 365 days for
an ordinary year and 366 days for a leap year.
1 interest period = 365 or 366 days

Note: A year is a leap year if it is divisible by 4 except when it is divisible by 100 but
not by 400.
Examples:
1996 (leap year)
1900 (not leap year)
2000 (leap year)
Also note: n must be expressed as
𝑥
𝑑𝑎𝑦𝑠: 𝑛 =
360 𝑜𝑟 365 𝑜𝑟 366
𝑥
𝑚𝑜𝑛𝑡ℎ𝑠: 𝑛 =
12
𝑥
𝑦𝑒𝑎𝑟𝑠: 𝑛 =
1
where x is the number of days/month/year

How to remember the number of days in a month using your fist:

Knuckle bone denotes 31 days and the gap between each knuckle bone is 30 days (or
28/29 for February).

Figure 5. Determining the Number of Days in a


Month using your Fist

Image source: Brightside

14
Examples:

1. A student borrows ₱50,000 from his aunt in order to finish his studies. His aunt
agrees to charge him simple interest at the rate of 5% per year. Suppose the student
wait two years and then repays the entire loan. How much will he have to repay?

Solution:

P = ₱50,000 ; i = 5% = 0.05 ; n = 2/1 = 2 ; F = ?

𝐹 = 𝑃(1 + 𝑖𝑛) = ₱50,000[1 + (0.05)(2)] = ₱55,000

2. Determine the ordinary simple interest on ₱10,000 for 9 months and 10 days if the
rate of interest is 12%.

Solution:

P = ₱10,000 ; i = 12% = 0.12 ; IO = ?

Based on banker’s year,


n = 9 months and 10 days = 9 (30) + 10 = 280 days

To find for the ordinary simple interest,


280
𝐼𝑜 = 𝑃𝑖𝑛 = (₱10,000)(0.12) ( ) = ₱933.33
360

3. Determine the ordinary and exact simple interest on ₱20,000 for the period from
January 25 to May 13, 2019 if the rate of simple interest is 18%.

Solution:

P = ₱20,000 ; i = 18% = 0.18 ; IO = ? ; IE = ?

2019 is not a leap year

First determine the number of days in the given period.


January 25 – 31 = 6 days (excluding January 25)
February = 28
March = 31
April = 30
May 1 – 13 = 13 (including May 13)
Total: = 108 days

To find for the ordinary simple interest,


108
𝐼𝑜 = 𝑃𝑖𝑛 = (₱20,000)(0.18) ( ) = ₱1,080
360

To find for the exact simple interest,


108
𝐼𝑒 = 𝑃𝑖𝑛 = (₱20,000)(0.18) ( ) = ₱1,065.2
365

4. Determine the ordinary and exact simple interest on ₱25,000 for the period from
January 16 to November 26, 2016 if the rate of simple interest is 22%.

15
Solution:

P = ₱25,000 ; i = 22% = 0.22 ; IO = ? ; IE = ?

2016 is a leap year

First determine the number of days in the given period.


January 16 – 31 = 15 days (excluding January 16)
February = 29
March = 31
April = 30
May = 31
June = 30
July = 31
August = 31
September = 30
October = 31
November 1 – 26 = 26 (including November 26)
Total: = 315 days

or
January 1 – 16 = 16 days (including January 16)
November 26 – 30 = 4 (excluding November 26)
December = 31
Total: = 51 days

366 – 51 = 315 days

To find for the ordinary simple interest,


315
𝐼𝑜 = 𝑃𝑖𝑛 = (₱25,000)(0.22) ( ) = ₱4,812.5
360

To find for the exact simple interest,


315
𝐼𝑒 = 𝑃𝑖𝑛 = (₱25,000)(0.22) ( ) = ₱4,733.6
366

5. If ₱3,000 is earned in 3 months on an investment of ₱120,000, what is the annual


rate of simple interest?

Solution:

I = ₱3,000 ; I = ₱120,000 ; n = 3/12 ; i = ?

𝐼 = 𝑃𝑖𝑛

𝐼 ₱3,000
𝑖= = = 0.1 = 10%
𝑃𝑛 (₱120,000) ( 3 )
12

16
Lesson 3: Discount

Discount on a negotiable paper is the difference between the present worth (the
amount received for the paper in cash) and the worth of the paper at some time in the
future (the face value of the paper or principal). Discount is interest paid in advance.

Discount = Future Worth – Present Worth

In symbols,

𝐷 = 𝐹−𝑃

The formulas to calculate for rate of discount and actual rate of interest are

𝐷
𝑑=
𝐹
𝐷
𝑖=
𝑃

The relationship of the two is given by

𝑖
𝑑=
1+𝑖
or

𝑑
𝑖=
1−𝑑
where:
D – discount
P – present worth or amount received
F – future worth or amount to be paid in future
d – rate of discount/interest or “banker’s discount”
i – actual rate of interest

Note that 𝑖 > 𝑑

Examples:

1. A man borrowed ₱5,000 from a bank and agreed to pay the loan at the end of 9
months. The bank discounted the loan and gave him ₱4,000 in cash. (a) What was the
rate of discount? (b) What was the rate of interest? (c) What was the rate of interest
for one year?

Solution:

P = ₱4,000 ; F = ₱5,000 ; d = ? ; i = ?

(a)
𝐷 = 𝐹 − 𝑃 = ₱5,000 − ₱4,000 = ₱1,000

𝐷 ₱1,000
𝑑= = = 0.2 = 20%
𝐹 ₱5,000

17
(b)
𝐷 ₱1,000
𝑖= = = 0.25 = 25%
𝑃 ₱4,000
or
𝑑 0.2
𝑖= = = 0.25 = 25%
1 − 𝑑 1 − 0.2
(c)

𝐼 𝐷 ₱1,000
𝑖= = = = 0.3333 = 33.33%
𝑃𝑛 𝑃𝑛 (₱4,000) ( 9 )
12

2. A man borrows ₱10,000 from a loan firm. The rate of simple interest is 15%, but the
interest is to be deducted from the loan at the time the money is borrowed. At the end
of one year he has to pay back ₱10,000. What is the actual rate of interest.

Solution:

F = ₱10,000 ; d = 15% = 0.15 ; i = ?

𝐷 = 𝐹−𝑃
𝐹𝑑 = 𝐹 − 𝑃
𝑃 = 𝐹 − 𝐹𝑛

𝑃 = 𝐹 − 𝐹𝑛 = ₱10,000 − 0.15(₱10,000 ) = ₱8,500

𝐷 ₱1,500
𝑖= = = 0.1765 = 17.65%
𝑃 ₱8,500

or you can use


𝑑 0.15
𝑖= = = 0.1765 = 17.65%
1 − 𝑑 1 − 0.15

Lesson 4: Cashflow Diagrams

A cashflow diagram is simply a graphical representation of cash flows drawn on a time


scale. Cash flow diagram for economic analysis problems is analogous to that of free
body diagram for mechanics problems.

 receipt (positive cashflow or cash inflow)

 disbursement (negative cashflow or cash outflow)

18
A loan of P100 at simple interest of 10% will become P150 after 5 years.

Lesson 5: Compound Interest and Time Value of Money

If in simple interest the interest acquires no interest itself, here in compound interest,
not only the principal accumulates interest but also the interest too. Hence calling
compound interest “interest on top of interest.”

In calculating compound interest, the interest for the current period is computed based
on the amount (principal plus interest up to the end of the previous period) at the
beginning of the current period.

Interest Principal at the Interest Earned Amount at End of Period


Period Beginning of During Period*
Period
1 P Pi P+Pi = P(1+i)
2 P(1+i) P(1+i) i P(1+i) + P(1+i) i =
P+Pi+Pi+Pi2 = P(1+2i+i2) =
P(1+i)2

19
3 P(1+i)2 P(1+i)2 i P(1+i)2 + P(1+i)2 i =
P1+P2i+Pi2 + Pi+P2i2+ Pi3 =
P(1+3i+3i2+i3) = P (1+i)3
⋯ ⋯ ⋯ ⋯
N P(1+i)n-1 P(1+i)n-1 i P(1+i)n

*Our n here is always 1, because the interval of interest period is 1. Hence, I = Pin = Pi(1) = Pi

Therefore, the future amount of compound interest after n interest periods is given by

𝐹 = 𝑃(1 + 𝑖)𝑛

The quantity (1 + 𝑖)𝑛 is called the “single payment compound amount factor” and is
designated by the function symbol F/P, i%, n (read as F given P at i percent in n interest
periods). Thus

𝐹 = 𝑃 (𝐹/𝑃, 𝑖%, 𝑛)

Figure 6. Cashflow diagram for Single Payment Compound Amount

From the formula above, we can also find the present worth amount (P) of a single
future sum (F) which will be received after n periods at an interest rate of i compounded
at the end of every interest period.

𝐹
𝑃=
(1 + 𝑖)𝑛

𝑃 = 𝐹(1 + 𝑖)−𝑛

𝑃 = 𝐹(𝑃/𝐹, 𝑖%, 𝑛)

The quantity (1 + 𝑖)−𝑛 is called the “single payment present worth factor” and is
designated by the function symbol P/F, i%, n (read as P given F at i percent in n interest
periods).

Figure 7. Cashflow diagram for Single Payment Present Worth Amount

20
Time Value of Money

Since money has the ability to earn interest, its value increases with time. If an investor
invests a sum of ₱1000 in a fixed deposit for five years with an interest rate of 15%
compounded annually, the accumulated amount at the end of every year will be as
shown in Table below.

Year End Interest (₱) Compound Amount (₱)


0 1000
1 150 1150
2 172.5 1322.5
3 198.4 1520.9
4 228.1 1749
5 262.4 2011.4

Formula used: 𝐹 = 𝑃(1 + 𝑖)𝑛

Since money increases in value as we move from the present to the future, it must
decrease in value as we move from the future to the present.

Rate of Interest

a. Nominal rate of interest, r – is the basic annual rate of interest


𝑟
𝑖=
𝑚
where:
i – rate of interest per interest period
r – nominal interest rate
m – no. of interest period per year; mode of compounding

Illustrative Example:
If the nominal rate of interest is 20% compounded quarterly, therefore

𝑟 20%
𝑖= = = 5% 𝑝𝑒𝑟 𝑝𝑒𝑟𝑖𝑜𝑑 = 5% 𝑝𝑒𝑟 𝑞𝑢𝑎𝑟𝑡𝑒𝑟
𝑚 4

b. Effective rate of interest, ie – is the actual or exact rate of interest earned on the
principal during a one-year period.
𝑟 𝑚
𝑖𝑒 = (1 + ) − 1
𝑚

Illustrative Example:
If ₱1.00 is invested at a nominal rate of interest is 20% compounded quarterly, after
one year this will become
𝑟 𝑚𝑁 0.2 4(1)
𝐹 = 𝑃(1 + 𝑖)𝑛 = 𝑃 (1 + ) = ₱1 (1 + ) = ₱1.2155
𝑚 4

The actual interest earned is


𝐼 = 𝐹 − 𝑃 = ₱1.2155 − ₱1.00 = ₱0.2155

Therefore, the actual rate of interest after one year is

21
₱0.2155
𝑖𝑒 = × 100 = 21.55%
₱1.00
Derivation:

For n = 1,
𝐼 =𝐹−𝑃

𝑟 𝑚
𝐼 = 𝑃 (1 + ) −𝑃
𝑚
𝑟 𝑚
𝐼 = 𝑃 [(1 + ) − 1]
𝑚
𝐼 𝑟 𝑚
= 𝑖𝑒 = (1 + ) − 1
𝑃 𝑚

Note:
𝑖𝑒 ≥ 𝑟
and
𝑖𝑒 = 𝑟 = 𝑖 𝑖𝑓 𝑚 = 1, 𝑛 = 𝑁

Number of interest periods, n


𝑛 = 𝑚𝑁
where:
N – number of years
m – number of interest period per year

 compounded annually, m = 1
 compounded semi-annually, m = 2
 compounded quarterly, m = 4
 compounded monthly, m = 12
 compounded bi-monthly, m = 6
 compounded bi-annually, m = 0.5
 compounded continuously, m = ∞

Compounded monthly means that the interest is computed at the end of every month.
There are 12 interest periods in a year if the interest is compounded monthly.

The general form of future worth in compound interest is

𝑟 𝑚𝑁
𝐹 = 𝑃 (1 + )
𝑚
where:
P – principal or present worth
r – nominal interest rate
m – mode of compounding ; no. of interest period per year
N – no. of years
F – accumulated amount or future worth

Examples:

1. Ma. Vina invests a sum of ₱5,000 in Metrobank at a nominal interest rate of 12% for
10 years. The compounding is quarterly. Find the maturity amount of the deposit after
10 years.

22
Solution:

P = ₱5,000 ; r = 12% = 0.12 ; N = 10 years ; m = 4 ; F = ?

𝑟 𝑚𝑁 0.12 4(10)
𝐹 = 𝑃 (1 + ) = ₱5,000 (1 + ) = ₱16,310.19
𝑚 4

2. A man wishes to have a future sum of ₱500,000 for his son’s education after 10
years from now. What is the single-payment that he should deposit now so that he gets
the desired amount after 10 years? The bank gives 15% interest rate compounded
annually.

Solution:

P = ₱500,000 ; r = 15% = 0.15 ; N = 10 years ; m = 1 ; F = ?

𝑟 𝑚𝑁
𝐹 = 𝑃 (1 + )
𝑚
𝐹
𝑃=
𝑟 𝑚𝑁
(1 + )
𝑚

₱500,000
𝑃=
0.15 1(10)
(1 + )
1

𝑃 = ₱123,592.35

3. How many years will be required for a sum of money to double, if the annual interest
rate is 10%, compounded quarterly?

Solution:

F=2P ; r = 10% = 0.1 ; m = 4 ; N = ?

𝑟 𝑚𝑁
𝐹 = 𝑃 (1 + )
𝑚

0.1 4𝑁
2𝑃 = 𝑃 (1 + )
4

2 = (1.025)4𝑁

Take logarithm of both sides


log(2) = log(1.025)4𝑁

log(2) = 4𝑁 log(1.025)

log(2)
𝑁=
4 log(1.025)

𝑁 = 7 𝑦𝑒𝑎𝑟𝑠

23
4. What is the effective annual interest rate if the nominal interest rate of 8% is
compounded (a) monthly, (b) semi-annually, (c) quarterly?

Solution:

r = 8% = 0.08 ; m = 12 , 2 , 4 ; ie = ?

(a)
𝑟 𝑚 0.08 12
𝑖𝑒 = (1 + ) − 1 = (1 + ) − 1 = 0.083 = 8.3%
𝑚 12
(b)
𝑟 𝑚 0.08 2
𝑖𝑒 = (1 + ) − 1 = (1 + ) − 1 = 0.0816 = 8.16%
𝑚 2
(c)
𝑟 𝑚 0.08 4
𝑖𝑒 = (1 + ) − 1 = (1 + ) − 1 = 0.0824 = 8.24%
𝑚 4

5. Suppose that the interest rate is 10% per year, compounded annually. What is the
minimum amount of money that would have to be invested for a two-year period in
order to earn ₱300 in interest?

Solution:

I = ₱300 ; r = 10% = 0.1 ; m = 1 ; n = 2 ; P = ?

𝑟 𝑚𝑁
𝐹 = 𝑃 (1 + )
𝑚
𝐹
𝑃=
𝑟 𝑚𝑁
(1 + )
𝑚
𝑃+𝐼
𝑃=
𝑟 𝑚𝑁
(1 + )
𝑚
𝑃 + 300
𝑃=
(1 + 0.1)2

1.21𝑃 = 𝑃 + 300

0.21𝑃 = 300

𝑃 = ₱1,428.57

6. Find the amount at the end of two years and seven months if ₱1,000 is invested at
8% compounded quarterly using simple interest for anytime less than a year interest
period.

24
Solution:

For compound interest: 𝑟 = 8% , 𝑁 = 2 𝑦𝑒𝑎𝑟𝑠


For simple interest: 𝑟 = 8% , 𝑁 = 7 𝑚𝑜𝑛𝑡ℎ𝑠

𝑟 𝑚𝑁 0.08 2(4)
𝐹1 = 𝑃 (1 + ) = ₱1,000 (1 + ) = ₱1,171.66
𝑚 4
7
𝐹2 = 𝐹1 (1 + 𝑖𝑛) = ₱1,171.66 [1 + (0.08) ( )] = ₱1,226.34
12

7. A ₱2,000 loan was originally made at 8% simple interest for 4 years. At the end of
this period the loan was extended for 3 years, without the interest being paid, but the
new interest rate was made 10% compounded, semi-annually. How much should the
borrower pay at the end of 7 years?

𝐹4 = 𝑃(1 + 𝑖𝑛) = ₱2,000 [1 + (0.08)(4)] = ₱2,640

𝑟 𝑚𝑁 0.1 2(3)
𝐹7 = 𝐹4 (1 + ) = ₱2,640 (1 + ) = ₱3,537.86
𝑚 2

Lesson 6: Discrete Compounding

If cash flow transactions occur quarterly, but interest is compounded monthly, we may
wish to calculate the effective interest rate on a quarterly basis.

To change the rate of interest per interest period to another, we have

25
𝑖𝑒1 = 𝑖𝑒2

𝑟1 𝑚1 𝑟2 𝑚2
(1 + ) − 1 = (1 + ) −1
𝑚1 𝑚2

𝑟1 𝑚1 𝑟2 𝑚2
(1 + ) = (1 + )
𝑚1 𝑚2

Example:

1. Find the nominal rate which if converted, quarterly could be used instead of 12%
compounded monthly.

Solution:

r1 = 12% ; m1 = 12 ; m1 = 4 ; r2 = ?

𝑟2 𝑚2 𝑟1 𝑚1
(1 + ) = (1 + )
𝑚2 𝑚1

𝑟2 4 0.12 12
(1 + ) = (1 + )
4 12

𝑟2 4 0.12 12
1+ = √(1 + )
4 12

𝑟2
= (1.01)3 − 1
4
𝑟2
= 0.0303
4

𝑟2 = 0.1212

𝑟2 = 12.12% 𝑐𝑜𝑚𝑝𝑜𝑢𝑛𝑑𝑒𝑑 𝑞𝑢𝑎𝑟𝑡𝑒𝑟𝑙𝑦

Lesson 7: Continuous Compounding

To be competitive on the financial market, or to entice potential depositors, some


financial institutions offer more frequent compounding. As the number of compounding
periods (m) becomes very large, the interest rate per compounding period (r/m)
becomes very small.

To calculate the effective annual interest rate for continuous compounding,

𝑖𝑒 = 𝑒 𝑟 − 1

To calculate for the future worth of money,

𝐹 = 𝑃𝑒 𝑟𝑁

26
To calculate for the present worth of money,

𝑃 = 𝐹𝑒 −𝑟𝑁
where:
ie – effective annual interest rate
r – nominal interest rate per year
F – accumulated amount or future worth
P – principal or present worth
N – no. of years
e = 2.71828…

Examples:

1. What effective annual interest rate corresponds to a nominal interest rate of 10%
per year, compounded continuously?

Solution:

r = 10% ; ie = ?

𝑖𝑒 = 𝑒 𝑟 − 1 = 𝑒 0.1 − 1 = 0.1052 = 10.52%

2. Determine the nominal interest rate corresponding to an effective interest rate of


10% per year, compounded continuously.

Solution:

ie = 10% ; r = ?
𝑖𝑒 = 𝑒 𝑟 − 1

𝑖𝑒 + 1 = 𝑒 𝑟
Take natural logarithm of both sides,

ln(𝑖𝑒 + 1) = ln 𝑒 𝑟

ln(𝑖𝑒 + 1) = r ln 𝑒
but ln 𝑒 = 1
ln(𝑖𝑒 + 1) = 𝑟

r = ln(𝑖𝑒 + 1) = ln(0.1 + 1) = 0.0953 = 9.53%

3. How much money must be deposited in a savings account so that ₱55,000 can be
withdrawn 12 years hence, if the interest rate is 9% per year, compounded
continuously, and if all the interest is allowed to accumulate?

Solution:

F = ₱55,000 ; r = 9% = 0.09 ; N = 12 ; P = ?

𝐹 = 𝑃𝑒 𝑟𝑁

𝐹
𝑃=
𝑒 𝑟𝑁

27
𝑃 = 𝐹𝑒 −𝑟𝑁

𝑃 = 𝐹𝑒 −𝑟𝑁 = ₱55,000 [𝑒 −(0.09)(12) ] = ₱18,677.75

Lesson 8: Concept of Equivalence

Just as we must convert fractions to common denominators in order to add them


together, we must convert cash flows to a common basis in order to compare their
value. One aspect of this basis is the choice of a single point in time at which to make
our calculations. The base period chosen as reference point is called the focal date.

When selecting a focal date at which to compare the value of alternative cash flows,
we commonly use either the present time or some point in the future. The choice of
the focal date often depends on the circumstances surrounding a particular decision,
or it may be chosen for convenience.

Examples:

1. A man bought a lot worth ₱1,000,000 if paid in cash. On the installment basis, he
paid a down payment of ₱200,000; ₱300,000 at the end of one year; ₱400,000 at the
end of three years and a final payment at the end of five years. What was the final
payment if the interest was 20% compounded annually?

Solution:

The amount to be paid after the down payment is

₱1,000,000 − ₱200,000 = ₱800,000

₱800,000
c

i = 20%
Focal 0 1 2 3 4 5
date

₱300,000
₱400,000
Q

Use today as the focal date. Then bring all the quantities into the focal date. We will
use the formula 𝑃 = 𝐹(1 + 𝑖)−𝑛 since the amounts should be expressed at present.

Σ 𝑖𝑛𝑓𝑙𝑜𝑤 = Σ 𝑜𝑢𝑡𝑓𝑙𝑜𝑤

₱ 800,000 = ₱300,000(1 + 0.2)−1 + ₱400,000(1 + 0.2)−3 + Q(1 + 0.2)−5

28
₱ 800,000 = ₱300,000(1.2)−1 + ₱400,000(1.2)−3 + Q(1.2)−5

₱ 800,000 − ₱300,000(0.8333) − ₱400,000(0.5787)


Q =
0.4019

Q = ₱792,560

Another Solution:

You can use also year 5 as focal date. Here, we will use the formula 𝐹 = 𝑃(1 + 𝑖)𝑛

₱800,000

i = 20%
0 1 2 3 4 5 Focal
date

₱300,000
₱400,000
Q

Σ 𝑖𝑛𝑓𝑙𝑜𝑤 = Σ 𝑜𝑢𝑡𝑓𝑙𝑜𝑤

₱ 800,000 (1 + 0.2)5 = ₱300,000(1 + 0.2)4 + ₱400,000(1 + 0.2)2 + Q

Q = ₱ 800,000(2.4883) − ₱300,000(2.0736) − ₱400,000(1.44)

Q = ₱792,560

Lesson 9: Effects of Inflation

Inflation is the increase in the prices for goods and services from one year to another,
thus decreasing the purchasing power of money.

𝐹 = 𝑃(1 + 𝑓)𝑛

where:
P – present cost of a commodity
F – future cost of the same commodity
f – annual inflation rate
n – number of years

29
In an inflationary economy, the buying power of money decreases as costs increase.

𝑃
𝐹=
(1 + 𝑓)𝑛

If interest is being compounded at the same time that inflation is occurring, the future
worth will be

𝑃(1 + 𝑖)𝑛 1+𝑖 𝑛


𝐹= = 𝑃( )
(1 + 𝑓)𝑛 1+𝑓
where:
P – present amount
F – future worth
i – annual interest rate
f – annual inflation rate
n – number of years

Examples:

1. An item presently costs ₱1,000. If inflation is at the rate of 8% per year, what will
be the cost of the item in two years?

Solution:

P = ₱1,000 ; f = 8% = 0.08 ; n = 2 ; F = ?

𝐹 = 𝑃(1 + 𝑓)𝑛 = ₱1,000(1 + 0.08)2 = ₱1,166.40

2. An economy is experiencing inflation at an annual rate of 8%. If this continues, what


will ₱1,000 be worth two years from now, in terms of today’s pesos?

Solution:

P = ₱1,000 ; f = 8% = 0.08 ; n = 2 ; F = ?

𝑃 ₱1,000
𝐹= 𝑛 = = ₱857.34
(1 + 𝑓) (1 + 0.08)2

3. A man invested P10,000 at an interest rate of 10% compounded annually. What will
be the final amount of his investment, in terms of today’s pesos, after five years, if
inflation remains the same at the rate of 8% per year?

Solution:

P = ₱10,000 ; i = 10% = 0.1 ; f = 8% = 0.08 ; n = 5 ; F = ?

1+𝑖 𝑛 1 + 0.1 5
𝐹 = 𝑃( ) = ₱10,000 ( ) = ₱10,960.86
1+𝑓 1 + 0.08

Lesson 10: Annuities

Annuity is a series of equal payments occurring at equal intervals of time.

30
Ordinary Annuity

Ordinary annuity is one where the payments are made at the end of each period.

Case 1: Uniform Payment Series Present Worth Amount (P required, A given)

The objective of this mode of investment is to find the present worth of an equal
payment made at the end of every interest period for n interest periods at an interest
rate of i compounded at the end of every interest period.

Figure 8. Cashflow diagram of uniform payment series present worth amount

The formula to calculate P is

1 − (1 + 𝑖)−𝑛
𝑃 = 𝐴[ ]
𝑖
or

𝑃 = 𝐴 (𝑃/𝐴, 𝑖%, 𝑛)

where:

P – present worth
A – annual equivalent payment
i – interest rate
n – no. of interest periods
1−(1+𝑖)−𝑛
– uniform series present worth factor
𝑖
(𝑃/𝐴, 𝑖%, 𝑛) – P given A at i percent in n interest periods

Case 2: Uniform Payment Series Capital Recovery Amount (A required, P given)

The objective of this mode of investment is to find the annual equivalent amount (A)
which is to be recovered at the end of every interest period for n interest periods for a
loan (P) which is sanctioned now at an interest rate of i compounded at the end of
every interest period.

31
Figure 9. Cashflow diagram of uniform payment series capital recovery amount

The formula to calculate A is

𝑖
𝐴 = 𝑃[ ]
1 − (1 + 𝑖)−𝑛
or

𝐴 = 𝑃 (𝐴/𝑃, 𝑖%, 𝑛)

where:
P – present worth (loan amount)
A – annual equivalent payment (recovery amount)
i – interest rate
n – no. of interest periods
𝑖
1−(1+𝑖) −𝑛 – capital recovery factor

(𝐴/𝑃, 𝑖%, 𝑛) – A given P at i percent in n interest periods

Case 3: Uniform Payment Series Compound Amount (F required, A given)

In this type of investment mode, the objective is to find the future worth of n equal
payments which are made at the end of every interest period till the end of the nth
interest period at an interest rate of i compounded at the end of each interest period.

Figure 10. Cashflow diagram of uniform payment series compound amount

The formula to calculate F is

(1 + 𝑖)𝑛 − 1
𝐹 = 𝐴[ ]
𝑖
or

𝐹 = 𝐴 (𝐹/𝐴, 𝑖%, 𝑛)

32
where:
A – equal amount deposited at the end of each interest period
n – no. of interest periods
i – rate of interest
F – single future amount
(1+𝑖)𝑛 −1
[ ] – uniform series compound amount factor
𝑖
(𝐹/𝐴, 𝑖%, 𝑛) – F given A at i percent in n interest periods

Case 4: Uniform Payment Series Sinking Fund (A required, F given)

In this type of investment mode, the objective is to find the equivalent amount (A) that
should be deposited at the end of every interest period for n interest periods to realize
a future sum (F) at the end of the nth interest period at an interest rate of i.

Figure 11. Cashflow diagram of uniform payment series sinking fund

The formula to get A is

𝑖
𝐴 = 𝐹[ ]
(1 + 𝑖)𝑛 − 1
or

𝐴 = 𝐹 (𝐴/𝐹, 𝑖%, 𝑛)

where:
A – equal amount to be deposited at the end of each interest period
n – no. of interest periods
i – rate of interest
F – single future amount at the end of the nth period
𝑖
𝑛 – sinking fund factor
(1+𝑖) −1
(𝐴/𝐹, 𝑖%, 𝑛) – A given F at i percent in n interest periods

Case 5. Uniform Arithmetic Gradient

The objective of this mode of investment is to find the annual equivalent amount of a
series with an amount A1 at the end of the first year and with an equal increment (G)
at the end of each of the following n – 1 years with an interest rate i compounded
annually.

33
Figure 12. Cashflow diagram of uniform arithmetic gradient

The formula to compute A is

(1 + 𝑖)𝑛 − 𝑖𝑛 − 1
𝐴 = 𝐴1 + 𝐺 [ ]
𝑖(1 + 𝑖)𝑛 − 𝑖
or

𝐴 = 𝐴1 + 𝐺 (𝐴/𝐺, 𝑖%, 𝑛)

This formula simplifies uniform arithmetic gradient into ordinary annuities discussed
above.

The equivalent cashflow diagram will be:

0 1 2 3 4 n

A A A A A

Now, you can get the present worth of uniform arithmetic gradient by using

1 − (1 + 𝑖)−𝑛
𝑃 = 𝐴[ ]
𝑖

Conversely, if it is required to find the future worth of uniform arithmetic gradient, you
can apply the formula

(1 + 𝑖)𝑛 − 1
𝐹 = 𝐴[ ]
𝑖

where:
A1 – amount to be deposited after one year

34
A – equivalent uniform amount to be deposited per year
n – no. of interest periods
i – rate of interest
G – uniform gradient amount
(1+𝑖)𝑛 −𝑖𝑛−1
– uniform gradient series factor
𝑖(1+𝑖)𝑛 −𝑖

Examples:

1. A person who is now 35 years old is planning for his retired life. He plans to invest
an equal sum of ₱10,000 at the end of every year for the next 25 years starting from
the end of the next year. The bank gives 20% interest rate, compounded annually. Find
the maturity value of his account when he is 60 years old.

Solution
A = ₱ 10,000 n = 25 years i = 20% F = ?

(1 + 𝑖)𝑛 − 1
𝐹 = 𝐴[ ]
𝑖

(1 + 0.2)25 − 1
𝐹 = ₱10,000 [ ]
0.2

𝐹 = ₱4,719,811

2. A company has to replace a present facility after 15 years at an outlay of ₱ 500,000.


It plans to deposit an equal amount at the end of every year for the next 15 years at
an interest rate of 18% compounded annually. Find the equivalent amount that must
be deposited at the end of every year for the next 15 years.

Solution
F = ₱ 500,000 n = 15 years i = 18% A = ?

35
𝑖
𝐴 = 𝐹[ ]
(1 + 𝑖)𝑛 − 1

0.18
𝐴 = ₱500,000 [ ]
(1 + 0.18)15 − 1

𝐴 = ₱ 8,200

3. A company wants to set up a reserve which will help the company to have an annual
equivalent amount of ₱ 1,000,000 for the next 20 years towards its employees’ welfare
measures. The reserve is assumed to grow at the rate of 15% annually. Find the single-
payment that must be made now as the reserve amount.

Solution
A = ₱ 1,000,000 i = 15% n = 20 years P = ?

1 − (1 + 𝑖)−𝑛
𝑃 = 𝐴[ ]
𝑖

1 − (1 + 0.15)−20
𝑃 = ₱ 1,000,000 [ ]
0.15

𝑃 = ₱ 6,259,300

4. A bank gives a loan to a company to purchase an equipment worth ₱ 1,000,000 at


an interest rate of 18% compounded annually. This amount should be repaid in 15
yearly equal installments. Find the installment amount that the company has to pay to
the bank.

Solution
P = ₱ 1,000,000 i = 18% n = 15 years A = ?

𝑖
𝐴 = 𝑃[ ]
1 − (1 + 𝑖)−𝑛

36
0.18
𝐴 = ₱ 1,000,000 [ ]
1 − (1 + 0.18)−15

𝐴 = ₱196,400

5. A person is planning for his retired life. He has 10 more years of service. He would
like to deposit 20% of his salary, which is ₱4,000, at the end of the first year, and
thereafter he wishes to deposit the amount with an annual increase of ₱500 for the
next 9 years with an interest rate of 15%. Find the total amount at the end of the 10th
year of the above series.
Solution:
A1 = ₱ 4,000 G = ₱ 500 i = 15% n = 10 years A = ? & F = ?

(1 + 𝑖)𝑛 − 𝑖𝑛 − 1
𝐴 = 𝐴1 + 𝐺 [ ]
𝑖(1 + 𝑖)𝑛 − 𝑖

(1 + 0.15)10 − (0.15)(10) − 1
𝐴 = ₱ 4,000 + ₱ 500 [ ]
0.15(1 + 0.15)10 − 0.15

𝐴 = ₱5,691.60

This is equivalent to paying an equivalent amount of ₱ 5,691.60 at the end of every


year for the next 10 years. The future worth sum of this revised series at the end of the
10th year is obtained as follows:

(1 + 𝑖)𝑛 − 1
𝐹 = 𝐴[ ]
𝑖

(1 + 0.15)10 − 1
𝐹 = ₱5,691.6 [ ]
0.15

𝐹 = ₱115,562.25

At the end of the 10th year, the compound amount of all his payments will be ₱
115,562.25.

6. A person is planning for his retired life. He has 10 more years of service. He would
like to deposit ₱ 8,500 at the end of the first year and thereafter he wishes to deposit
the amount with an annual decrease of ₱ 500 for the next 9 years with an interest rate
of 15%. Find the total amount at the end of the 10th year of the above series.

37
Solution:
A1 = ₱ 8,500 G = – ₱ 500 i = 15% n = 10 years A = ? & F = ?

(1 + 𝑖)𝑛 − 𝑖𝑛 − 1
𝐴 = 𝐴1 + 𝐺 [ ]
𝑖(1 + 𝑖)𝑛 − 𝑖

(1 + 0.15)10 − (0.15)(10) − 1
𝐴 = ₱ 8,500 − ₱ 500 [ ]
0.15(1 + 0.15)10 − 0.15

𝐴 = ₱6,808.40

This is equivalent to paying an equivalent amount of ₱ 6,808.40 at the end of every


year for the next 10 years.
The future worth sum of this revised series at the end of the 10th year is obtained as
follows:

(1 + 𝑖)𝑛 − 1
𝐹 = 𝐴[ ]
𝑖

(1 + 0.15)10 − 1
𝐹 = ₱6,808.4 [ ]
0.15

𝐹 = ₱138,237.75

At the end of the 10th year, the compound amount of all his payments is ₱ 138,237.75.

More Examples:

7. What is the present worth of ₱500 deposited at the end of every three months for 6
years if the interest rate is 12% compounded semi-annually?

Solution:

A = ₱500 ; N = 6 years ; r = 12% = 0.12 ; P = ?

Convert first the semi-annual compounding into quarterly.

Solving for the interest rate per quarter,

𝑟1 𝑚1 𝑟2 𝑚2
(1 + ) = (1 + )
𝑚1 𝑚2

38
𝑟1 4 0.12 2
(1 + ) = (1 + )
4 2
𝑟1 4
1+ = √(1.06)2
4
𝑟1
= 1.0296 − 1
4
𝑟1
= 0.0296
4

𝑖1 = 2.96% 𝑝𝑒𝑟 𝑞𝑢𝑎𝑟𝑡𝑒𝑟

Now, to find for the present worth

1 − (1 + 𝑖)−𝑛
𝑃 = 𝐴[ ]
𝑖

1 − (1 + 0.0296)−4(6)
𝑃 = ₱500 [ ]
0.0296

𝑃 = ₱8,504

8. A chemical engineer wishes to set up a special fund by making uniform semi-annual


end-of-period deposits for 20 years. The fund is to provide ₱100,000 at the end of each
of the last five years of the 20-year period. If interest is 8% compounded semiannually,
what is the required semiannual deposit to be made?

Solution:

P100,000 (F/A, 8.16%, 5)

P100,000 P100,000 P100,000 P100,000 P100,000

15 16 17 18 19 20 Year

0 1 2 30 31 32 33 34 35 36 37 38 39 40 Period

A A A A A A A A A A A A A

A (F/A, 4%, 40)

For the deposits:

8%
𝑖= = 4%
2

39
For withdrawals (convert semi-annual rate to annual rate):

0.08 2 𝑟 1
(1 + ) = (1 + )
2 1

1.0816 = 1 + 𝑖

𝑖 = 0.0816

𝑖 = 8.16%

Using 20 years from today as the focal date:

(1+𝑖)𝑛 −1
(Note: we are going to use the formula 𝐹 = 𝐴 [ ] here).
𝑖

Σ 𝑖𝑛𝑓𝑙𝑜𝑤 (𝑑𝑒𝑝𝑜𝑠𝑖𝑡𝑠) = Σ 𝑜𝑢𝑡𝑓𝑙𝑜𝑤 (𝑤𝑖𝑡ℎ𝑑𝑟𝑎𝑤𝑎𝑙𝑠)

(1 + 0.04)40 − 1 (1 + 0.0816)5 − 1
𝐴[ ] = ₱100,000 [ ]
0.04 0.0816

𝐴 = ₱6,193.39

9. Using a compound interest of 8%, find the equivalent uniform annual cost for a
proposed machine that has a first cost of ₱100,000 an estimated salvage value of
₱20,000 and an estimated economic life of 8 years. Annual maintenance will amount
to ₱2,000 a year and periodic overhaul costing ₱6,000 each will occur at the end of
the second and fourth year.

Solution:

40
Let A = the equivalent uniform of annual cost

Using today as the focal date:

Σ 𝐸𝑞𝑢𝑖𝑣𝑎𝑙𝑒𝑛𝑡 𝑢𝑛𝑖𝑓𝑜𝑟𝑚 𝑎𝑛𝑛𝑢𝑎𝑙 𝑐𝑜𝑠𝑡


= Σ 𝑡𝑜𝑡𝑎𝑙 𝑐𝑜𝑠𝑡 (𝑜𝑢𝑡𝑓𝑙𝑜𝑤) − Σ 𝑠𝑎𝑙𝑣𝑎𝑔𝑒 𝑣𝑎𝑙𝑢𝑒 (𝑖𝑛𝑓𝑙𝑜𝑤)

1 − (1.08)−8 1 − (1.08)−8
𝐴[ ] = 100,000 + 6,000(1.08)−2 + 6,000(1.08)−4 + 2,000 [ ]
0.08 0.08

−20,000(1.08)−8

𝐴 = ₱19,183

1−(1+𝑖)−𝑛
(Note: we use the formulas 𝑃 = 𝐴 [ ] and 𝑃 = 𝐹(1 + 𝑖)−𝑛 here).
𝑖

Deferred Annuity

A deferred annuity is one where the first payment is made several periods after the
beginning of the annuity.

Figure 13. Cashflow Diagram for Deferred Annuity

To calculate P when A is given:

1 − (1 + 𝑖)−𝑛
𝑃=𝐴 [ ] (1 + 𝑖)−𝑀
𝑖

where:
P – present worth
A – equal amount to be deposited at the end of each interest period
i – interest rate
n – no. of interest periods
M – no. of deferred periods

*The focal date of this equation is today (or year 0).

41
Examples:

1. On the day his grandson was born, a man deposited to a trust company a sufficient
amount of money so that the boy could receive 5 annual payments of ₱10,000 each
for his college tuition fees, starting with his 18 th birthday. Interest at the rate of 12% per
annum was to be paid on all amounts on deposit. There was also a provision that the
grandson could elect to withdraw no annual payments and receive a single lump
amount on his 25th birthday. The grandson chose this option. (a) How much did the
boy receive as the single payment? (b) How much did the grandfather deposit?

Solution:

A = ₱10,000 ; n = 5 (18th to 22nd birthday) ; M = 3 (23rd to 25th birthday) or M = 17 (0 to


17th birthday) ; i = 12%

Let P = the amount deposited


X = the amount withdrawn

The P10,000 suppose withdrawals are represented by broken lines, since they did not
actually occur. Three separate cash flow diagrams can be drawn.

42
(a) Using 25 years of age as the focal date:

(1 + 𝑖)𝑛 − 1
𝐴[ ] (1 + 𝑖)𝑀 = 𝑋
𝑖

(1 + 0.12)5 − 1
₱10,000 [ ] (1 + 0.12)3 = 𝑋
0.12

₱89,250 = X

(b) Using today as the focal date:

1 − (1 + 𝑖)−𝑛
𝑃=𝐴 [ ] (1 + 𝑖)−𝑀
𝑖

1 − (1 + 0.12)−5
𝑃 = ₱10,000 [ ] (1 + 0.12)−17
0.12

𝑃 = ₱5,250

or you can use lump sum found on (a) then bring it to the present

𝑃 = 𝐹(1 + 𝑖)−𝑛

𝑃 = 𝑋(1 + 𝑖)−𝑛

𝑃 = ₱89,250 (1 + 0.12)−25

𝑃 = ₱5,250

The other good focal dates are 17 and 22 years from today.

43
2. If ₱10,000 is deposited each year for 9 years, how much annuity can a person get
annually from the bank every year for 8 years starting 1 year after the 9 th deposit is
made. Cost of money is 14%.

Solution:

Using today as the focal date,

1 − (1 + 𝑖)−𝑛 −𝑀
1 − (1 + 𝑖)−𝑛
𝐴[ ] (1 + 𝑖) = 𝐴 [ ]
𝑖 𝑖

1 − (1 + 0.14)−8 1 − (1 + 0.14)−9
𝐴 [ ] (1 + 0.14)−9 = ₱10,000 [ ]
0.14 0.14

A = ₱34,675

Another solution:

Using year 17 as the focal date,

44
(1 + 𝑖)𝑛 − 1 (1 + 𝑖)𝑛 − 1
𝐴[ ] = 𝐴[ ] (1 + 𝑖)𝑀
𝑖 𝑖

(1 + 0.14)8 − 1 (1 + 0.14)9 − 1
𝐴[ ] = ₱10,000 [ ] (1 + 0.14)8
0.14 0.14

𝐴 = ₱34,675

You can also use year 9 as focal date.

Annuity Due

An annuity due is one where the payments are made at the beginning of each period.

Figure 14. Cashflow Diagram for Annuity Due

To calculate P when A is given:

1 − (1 + 𝑖)−(𝑛−1)
𝑃 = 𝐴 + 𝐴[ ]
𝑖

where:
P – present worth
A – equal amount to be deposited at the beginning of each interest period
i – interest rate
n – no. of interest periods

To calculate F when A is given:

(1 + 𝑖)𝑛+1 − 1
𝐹 = 𝐴[ ]−𝐴
𝑖

where:
F – future worth
A – equal amount to be deposited at the beginning of each interest period
i – interest rate
n – no. of interest periods

45
Examples:

1. A man bought an equipment costing ₱60,000 payable in 12 quarterly payments,


each instalment payable at the beginning of each period. The rate of interest is 24%
compounded quarterly. What is the amount of each payment?

Solution:

P = ₱60,000 ; n =12 ; r = 24%, m = 4 ; A = ?

1 − (1 + 𝑖)−(𝑛−1)
𝑃 = 𝐴 + 𝐴[ ]
𝑖

1 − (1 + 𝑖)−(𝑛−1)
𝑃 = 𝐴 [1 + ( )]
𝑖

0.24 −(12−1)
1 − (1 + )
₱60,000 = 𝐴 [1 + ( 4 )]
0.24
4

₱60,000 = 𝐴(8.8869)

₱6751.51 = 𝐴

Perpetuity

Perpetuity is an annuity in which the payments continue indefinitely.

Figure 15. Cashflow Diagram for Perpetuity

To calculate P when A is given:

𝐴
𝑃=
𝑖

where:
P – present worth
A – equal amount to be deposited
i – interest rate

46
Examples:

What amount of money invested today at 15% interest can provide the following
scholarships: ₱30,000 at the end of each year for 6 years; ₱40,000 for the next 6 years
and ₱50,000 thereafter?

Solution:

Using today as the focal date:

𝑇𝑜𝑡𝑎𝑙 𝑝𝑟𝑒𝑠𝑒𝑛𝑡 𝑣𝑎𝑙𝑢𝑒 𝑜𝑓 𝑖𝑛𝑣𝑒𝑠𝑡𝑚𝑒𝑛𝑡


= 𝑜𝑟𝑑𝑖𝑛𝑎𝑟𝑦 𝑎𝑛𝑛𝑢𝑖𝑡𝑦 𝑜𝑓 ₱30,000 + 𝑑𝑒𝑓𝑒𝑟𝑟𝑒𝑑 𝑎𝑛𝑛𝑢𝑖𝑡𝑦 𝑜𝑓 ₱40,000
+ 𝑑𝑒𝑓𝑒𝑟𝑟𝑒𝑑 𝑝𝑒𝑟𝑝𝑒𝑡𝑢𝑖𝑡𝑦 𝑜𝑓 ₱50,000

1 − (1 + 𝑖)−𝑛 1 − (1 + 𝑖)−𝑛 𝐴
𝑃=𝐴 [ ]+𝐴 [ ] (1 + 𝑖)−𝑀 + (1 + 𝑖)−𝑀
𝑖 𝑖 𝑖

1 − (1 + 0.15)−6 1 − (1 + 0.15)−6
𝑃 = ₱30,000 [ ] + ₱40,000 [ ] (1 + 0.15)−6
0.15 0.15
₱50,000
+ (1.15)−12
0.15

𝑃 = 241,277

Lesson 11: Capitalized Cost

One of the most important applications of perpetuity is in capitalized cost. The


capitalized cost of any property is the sum of the first cost and the present worth of all
costs of replacement, operation, and maintenance for a long time or forever.

47
Case 1. No replacement, only maintenance and or operation every period.

𝐶𝑎𝑝𝑖𝑡𝑎𝑙𝑖𝑧𝑒𝑑 𝐶𝑜𝑠𝑡
= 𝐹𝑖𝑟𝑠𝑡 𝑐𝑜𝑠𝑡
+ 𝑃𝑟𝑒𝑠𝑒𝑛𝑡 𝑤𝑜𝑟𝑡ℎ 𝑜𝑓 𝑝𝑒𝑟𝑝𝑒𝑡𝑢𝑎𝑙 𝑜𝑝𝑒𝑟𝑎𝑡𝑖𝑜𝑛 𝑎𝑛𝑑/𝑜𝑟 𝑚𝑎𝑖𝑛𝑡𝑒𝑛𝑎𝑛𝑐𝑒

𝐶𝑎𝑝𝑖𝑡𝑎𝑙𝑖𝑧𝑒𝑑 𝐶𝑜𝑠𝑡 = 𝐶𝑜 + 𝑃

Example:

Determine the capitalized cost of a structure that requires an initial investment of


₱1,500,000 and an annual maintenance of ₱150,000. Interest is 15%.

Solution:

𝐶𝑎𝑝𝑖𝑡𝑎𝑙𝑖𝑧𝑒𝑑 𝐶𝑜𝑠𝑡 = 𝐹𝑖𝑟𝑠𝑡 𝑐𝑜𝑠𝑡 + 𝑃

𝐴
𝐶𝑎𝑝𝑖𝑡𝑎𝑙𝑖𝑧𝑒𝑑 𝐶𝑜𝑠𝑡 = 𝐶𝑜 +
𝑖
₱150,000
𝐶𝑎𝑝𝑖𝑡𝑎𝑙𝑖𝑧𝑒𝑑 𝐶𝑜𝑠𝑡 = ₱1,500,000 +
0.15

𝐶𝑎𝑝𝑖𝑡𝑎𝑙𝑖𝑧𝑒𝑑 𝐶𝑜𝑠𝑡 = ₱2,500,000

Case 2. Replacement only, no maintenance and/or operation.

𝐶𝑎𝑝𝑖𝑡𝑎𝑙𝑖𝑧𝑒𝑑 𝐶𝑜𝑠𝑡 = 𝐹𝑖𝑟𝑠𝑡 𝑐𝑜𝑠𝑡 + 𝑃𝑟𝑒𝑠𝑒𝑛𝑡 𝑤𝑜𝑟𝑡ℎ 𝑜𝑓 𝑝𝑒𝑟𝑝𝑒𝑡𝑢𝑎𝑙 𝑟𝑒𝑝𝑙𝑎𝑐𝑒𝑚𝑒𝑛𝑡

𝐶𝑜 − 𝐶𝑛
𝐶𝑎𝑝𝑖𝑡𝑎𝑙𝑖𝑧𝑒𝑑 𝐶𝑜𝑠𝑡 = 𝐶𝑜 +
(1 + 𝑖)𝑛 − 1

Example:

A new engine was installed by a textile plant at a cost of ₱300,000 and projected to
have a useful life of 15 years. At the end of its useful life, it is estimated to have a
salvage value of ₱30,000. Determine its capitalized cost if interest is 18% compounded
annually.

48
Solution:

𝐶𝑎𝑝𝑖𝑡𝑎𝑙𝑖𝑧𝑒𝑑 𝐶𝑜𝑠𝑡 = 𝐹𝑖𝑟𝑠𝑡 𝑐𝑜𝑠𝑡 + 𝑃𝑟𝑒𝑠𝑒𝑛𝑡 𝑤𝑜𝑟𝑡ℎ 𝑜𝑓 𝑝𝑒𝑟𝑝𝑒𝑡𝑢𝑎𝑙 𝑟𝑒𝑝𝑙𝑎𝑐𝑒𝑚𝑒𝑛𝑡

𝐶𝑜 − 𝐶𝑛
𝐶𝑎𝑝𝑖𝑡𝑎𝑙𝑖𝑧𝑒𝑑 𝐶𝑜𝑠𝑡 = 𝐶𝑜 +
(1 + 𝑖)𝑛 − 1

₱300,000 − ₱30,000
𝐶𝑎𝑝𝑖𝑡𝑎𝑙𝑖𝑧𝑒𝑑 𝐶𝑜𝑠𝑡 = ₱300,000 +
(1 + 0.18)15 − 1

𝐶𝑎𝑝𝑖𝑡𝑎𝑙𝑖𝑧𝑒𝑑 𝐶𝑜𝑠𝑡 = ₱324,604

Case 3. Replacement, maintenance and/or operation every period

𝐶𝑎𝑝𝑖𝑡𝑎𝑙𝑖𝑧𝑒𝑑 𝐶𝑜𝑠𝑡
= 𝐹𝑖𝑟𝑠𝑡 𝑐𝑜𝑠𝑡
+ 𝑃𝑟𝑒𝑠𝑒𝑛𝑡 𝑤𝑜𝑟𝑡ℎ 𝑜𝑓 𝑐𝑜𝑠𝑡 𝑜𝑓 𝑝𝑒𝑟𝑝𝑒𝑡𝑢𝑎𝑙 𝑜𝑝𝑒𝑟𝑎𝑡𝑖𝑜𝑛 𝑎𝑛𝑑
/𝑜𝑟 𝑚𝑎𝑖𝑛𝑡𝑒𝑛𝑎𝑛𝑐𝑒 + 𝑃𝑟𝑒𝑠𝑒𝑛𝑡 𝑤𝑜𝑟𝑡ℎ 𝑜𝑓 𝑐𝑜𝑠𝑡 𝑜𝑓 𝑝𝑒𝑟𝑝𝑒𝑡𝑢𝑎𝑙 𝑟𝑒𝑝𝑙𝑎𝑐𝑒𝑚𝑒𝑛𝑡

𝐶𝑜 − 𝐶𝑛
𝐶𝑎𝑝𝑖𝑡𝑎𝑙𝑖𝑧𝑒𝑑 𝐶𝑜𝑠𝑡 = 𝐶𝑜 + 𝑃 +
(1 + 𝑖)𝑛 − 1

Example:

Determine the capitalized cost of a research laboratory which requires ₱5,000,000 for
original construction; ₱100,000 at the end of every year for the first 6 years and then
₱120,000 each year thereafter for operating expenses, and ₱500,000 every 5 years
for replacement of equipment with interest at 12% per annum?

Solution:

49
From the above figure, Q (we use P instead) is the present worth of cost of perpetual
operation. Here, you need to bring the worth of ₱100,000 and ₱120,000 annuities to
present. Take note however that the ₱100,000 annuity is an ordinary annuity while
₱120,000 is a form of deferred perpetuity.

𝐶𝑜 −𝐶𝑛
From the above figure, X (we use the formula here) is the present worth of cost
(1+𝑖)𝑛 −1
of perpetual replacement.

From the formula,


𝐶𝑜 − 𝐶𝑛
𝐶𝑎𝑝𝑖𝑡𝑎𝑙𝑖𝑧𝑒𝑑 𝐶𝑜𝑠𝑡 = 𝐶𝑜 + 𝑃 +
(1 + 𝑖)𝑛 − 1

1 − (1 + 𝑖)−𝑛 𝐴 𝐶𝑜 − 𝐶𝑛
𝐶𝑎𝑝𝑖𝑡𝑎𝑙𝑖𝑧𝑒𝑑 𝐶𝑜𝑠𝑡 = 𝐶𝑜 + (𝐴 [ ] + (1 + 𝑖)−𝑀 ) +
𝑖 𝑖 (1 + 𝑖)𝑛 − 1

50
𝐶𝑎𝑝𝑖𝑡𝑎𝑙𝑖𝑧𝑒𝑑 𝐶𝑜𝑠𝑡
1 − (1 + 0.12)−6 ₱120,000
= ₱5,000,000 + ₱100,000 [ ]+ (1 + 0.12)−6
0.12 0.12
₱500,000
+
(1 + 0.12)5 − 1

𝐶𝑎𝑝𝑖𝑡𝑎𝑙𝑖𝑧𝑒𝑑 𝐶𝑜𝑠𝑡 = ₱5,000,000 + ₱917,740 + ₱655,910

𝐶𝑎𝑝𝑖𝑡𝑎𝑙𝑖𝑧𝑒𝑑 𝐶𝑜𝑠𝑡 = ₱6,753,650

Lesson 12: Amortization

Amortization is any method of repaying debt, the principal and interest included,
usually by a series of equal payments at equal interval of time.

Examples:

1. A debt of ₱5,000 with interest at 12% compounded semiannually is to be amortized


by equal semi-annual payments over the next 3 years, the first due in 6 months. Find
the semiannual payment and construct an amortization schedule.

Solution:
12%
𝑖= = 6%
2

𝑖 0.06
𝐴 = 𝑃[ ] = ₱5,000 [ ] = ₱1,016.82
1 − (1 + 𝑖)−𝑛 1 − (1 + 0.06)−2(3)

Table 1. Amortization Schedule

Outstanding
Interest due at Principal repaid
Period principal at the Payment
end of period at end of period
beginning of period
1 ₱ 5,000.00 ₱ 300.00 ₱ 1,016.82 ₱ 716.82
2 4,283.18 256.99 1,016.82 759.83
3 3,523.35 211.40 1,016.82 805.42
4 2,717.93 163.08 1,016.82 853.74
5 1,867.19 111.85 1,016.82 904.97
6 959.22 57.55 1,016.82 959.27
TOTAL ₱ 1,100.87 ₱ 6,100.92 ₱ 5,000.05

51
How does third column values computed?

For third column, second row:


𝐼 =𝐹−𝑃

𝐼 = 𝑃(1 + 𝑖)𝑛 − 𝑃

𝐼 = ₱ 5,000(1 + 0.06)1 − ₱ 5,000

𝐼 = ₱300

For third column, third row:

𝐼 = ₱ 4,283.18(1 + 0.06)1 − ₱ 4,283.18

𝐼 = ₱256.99
And so on…

How does fifth column values computed?

𝐴𝑚𝑜𝑢𝑛𝑡 𝑝𝑎𝑖𝑑 𝑜𝑛 𝑡ℎ𝑒 𝑝𝑟𝑖𝑛𝑐𝑖𝑝𝑎𝑙 = 𝐴 − 𝐼

𝐴𝑚𝑜𝑢𝑛𝑡 𝑝𝑎𝑖𝑑 𝑜𝑛 𝑡ℎ𝑒 𝑝𝑟𝑖𝑛𝑐𝑖𝑝𝑎𝑙 = ₱1,016.82 − ₱300

𝐴𝑚𝑜𝑢𝑛𝑡 𝑝𝑎𝑖𝑑 𝑜𝑛 𝑡ℎ𝑒 𝑝𝑟𝑖𝑛𝑐𝑖𝑝𝑎𝑙 = ₱716.82

And so on…

How does second column values computed (except for the first entry, i.e. ₱ 5,000)?

𝑂𝑢𝑡𝑠𝑡𝑎𝑛𝑑𝑖𝑛𝑔 𝑝𝑟𝑖𝑛𝑐𝑖𝑝𝑎𝑙 = 𝑃𝑟𝑖𝑛𝑐𝑖𝑝𝑎𝑙 − 𝐴𝑚𝑜𝑢𝑛𝑡 𝑝𝑎𝑖𝑑 𝑜𝑛 𝑡ℎ𝑒 𝑝𝑟𝑖𝑛𝑐𝑖𝑝𝑎𝑙

𝑂𝑢𝑡𝑠𝑡𝑎𝑛𝑑𝑖𝑛𝑔 𝑝𝑟𝑖𝑛𝑐𝑖𝑝𝑎𝑙 = ₱ 5,000 − ₱716.82

𝑂𝑢𝑡𝑠𝑡𝑎𝑛𝑑𝑖𝑛𝑔 𝑝𝑟𝑖𝑛𝑐𝑖𝑝𝑎𝑙 = ₱ 4,283.18

And so on…

2. A debt of ₱ 10,000 with interest at the rate of 20% compounded semiannually is to


be amortized by 5 equal payments at the end of each 6 months, the first payment is to
be made after 3 years. Find the semiannual payment and construct the amortization
schedule.

Solution:

20%
𝑖= = 10%
2

52
1 − (1 + 𝑖)−𝑛
𝑃 = 𝐴[ ] (1 + 𝑖)−𝑀
𝑖

𝑃 ₱10,000
𝐴= = = ₱4,248.50
1 − (1+ 𝑖)−𝑛 1 − (1 + 0.1)−5
[ ] (1 + 𝑖)−𝑀 [ ] (1 + 0.1) −5
𝑖 0.1

Table 2. Amortization Schedule

Outstanding
Interest due at Principal repaid
Period principal at the Payment
end of period at end of period
beginning of period
1 ₱ 10,000.00 ₱ 1,000.00
2 11,000.00 1,100.00
3 12,100.00 1,210.00
4 13,310.00 1,331.00
5 14,641.00 1,464.10
6 16,105.10 1,610.51 ₱4,248.50 ₱ 2,637.99
7 13,467.11 1,346.71 4,248.50 2,901.79
8 10,565.32 1,056.53 4,248.50 3,191.97
9 7,373.35 737.34 4,248.50 3,511.16
10 3,862.19 386.22 4,248.50 3,862.28
TOTAL ₱ 11,242.41 ₱ 21,242.50 ₱ 16,105.19

53
CHAPTER EXERCISES

Objective Test 2

1. What is defined as the interest on a loan or principal that is based only on the original
amount of the loan or principal?

a. effective rate of interest b. nominal rate of interest


c. compound interest d. simple interest

2. Under ordinary simple interest, how many days is one year?

a. 300 b. 360 c. 365 d. 366

3. One banker’s years is equivalent to ______ days.

a. 300 b. 360 c. 365 d. 366

4. What refers to the cumulative effect of elapsed time on the money value of an event,
based on the earning power of equivalent invested funds capital should or will earn?

a. present worth factor b. interest rate


c. time value of money D. yield

5. What refers to the amount of money paid for the use of borrowed capital?

a. interest b. rate of interest c. simple interest d. principal

6. What is defined as the interest of loan or principal which is based not only on the
original amount on the loan or principal but the amount of the loaned or principal plus
the previous accumulated interest?

a. effective rate of interest b. nominal rate of interest


c. compound interest d. simple interest

7. The difference between the present and future worth of money at some time in the
future is called ____________.

a. discount b. deduction c. inflation d. depletion

8. A uniform series of payment occurring at equal interval of time is called ________.

a. amortization b. annuity c. bond d. depreciation

9. What is the type of annuity where the payments are made at the end of each period
starting from the first period?

a. ordinary annuity b. perpetuity c. deferred annuity d. annuity due

10. What is the type of annuity where the payments are made at the beginning of each
period starting from the first period?

54
a. ordinary annuity b. perpetuity c. deferred annuity d. annuity due

11. What is the type of annuity that does not have a fixed time span but continues
indefinitely or forever?

a. ordinary annuity b. perpetuity c. deferred annuity d. annuity due

12. What is the type of annuity where the first payment does not begin until some later
date in the cash flow?

a. ordinary annuity b. perpetuity c. deferred annuity d. annuity due

13. Which is NOT an essential element of an ordinary annuity?

a. The amounts of all payments are equal.


b. The payments are made at equal interval of time.
c. The first payment is made at the beginning of the first period.
d. Compound interest is paid on all amounts in the annuity.

14. What refers to the present worth of cost associated with an asset for an infinite
period of time?

a. annual cost b. increment cost c. capitalized cost d. operating cost

15. Capitalized cost of a project is also known as ____________.

a. infinite cost b. life cycle cost c. life cost d. project cost

16. What is the factor name for the formula (1+i)^-n?

a. uniform gradient series b. single payment compound amount


c. single payment present worth d. uniform series present worth

17. It is the process of paying off debt through regular payments over time.

a. amortization b. annuity c. bond d. depreciation


𝑖
18. The factor is called
1−(1+𝑖)−𝑛

a. capital recovery factor b. sinking fund factor


c. uniform series compound amount factor d. uniform series present worth factor
(1+𝑖)𝑛 −1
19. The factor is called
𝑖

a. capital recovery factor b. sinking fund factor


c. uniform series compound amount factor d. uniform series present worth factor

20. __________ is the sustained increase in the price of goods and services in an
economy over a period of time.

a. discount b. deflation c. inflation d. depletion

55
Problem Solving 2

Simple Interest

1. How much interest would be due at the end of one year on a loan of ₱10,000 if the
interest is 12% per year? Ans. ₱1,200

2. How long will it take for an investment of ₱5,000 to grow to ₱7,500, if it earns 10%
simple interest per year? Ans. 5 years

3. What is the annual interest rate on a ₱150,000 loan in which all interest is paid at
the end of the year, and a total of ₱164,250 must be repaid at the end of the year?
Ans. 9.5%

4. What is the annual rate of simple interest if ₱265 is earned in four months on an
investment of ₱15,000? Ans. 5.3%

5. If you borrow money from your fried at 12% simple interest, determine the present
worth of ₱20,000 which is due at the end of 9 months. Ans. ₱18,348.62

6. Determine the ordinary and exact simple interest on ₱30,000 for the period from
January 15 to June 20, 2019 if the rate of simple interest is 15%. Ans. IO = ₱1,950 ; IE
= ₱1,923.29

7. Determine the exact simple interest on P5,000 for the period from Jan. 15 to Nov.
28, 1992, if the rate of interest is 22%. Ans. ₱955.74

Discount

8. A man borrowed from a bank. He received from the bank ₱13,420 and promise to
repay ₱15,000 at the end of 9 months. Determine the simple interest rate and the
corresponding discount rate or often referred to as the “Banker’s discount.” Ans.
11.7%, 10.53%

Compound Interest

9. What is the effective annual interest rate if the nominal interest rate is 6%,
compounded monthly? Ans. 6.1678% per year

10. A bank pays 6% interest per year, compounded quarterly. To what amount will a
₱50,000 deposit grow if left in that bank for 10 years? Ans. ₱90,700.9

11. What amount of money is equivalent to receiving ₱8,000 three years from today, if
the interest rate is 8% per year, compounded semiannually? Ans. ₱6,322.52

12. A person deposits ₱2000 in a savings account. If all of the money is allowed to
accumulate, how much will the person have at the end of 5 years, given a nominal
interest rate of 6%, compounded (a) annually? (b) quarterly? (c) monthly? (d) daily?
Ans. (a) ₱2676.40; (b) ₱2693.80; (c) ₱2697.80; (d) ₱2699.65

13. A man wishes his son to receive ₱200,000 ten years from now. What amount
should he invest if it will earn interest of 10% compounded annually during the first 5
years and 12% compounded quarterly during the next 5 years? Ans. ₱68,758.67

56
14. By the conditions of a will, the sum of ₱25,000 is left to a girl to be held in trust by
her guardian until it amounts to ₱45,000. When will the girl receive the money if the
fund is invested at 8% compounded quarterly? Ans. 7.42 years

15. At a certain interest rate compounded semiannually, ₱5,000 will amount to ₱20,000
after 10 years. What is the amount at the end of 15 years? Ans. ₱40,029.72

16. A savings bank offers ₱1000 certificates of deposit. Each certificate can be
redeemed for ₱2000 after 8.5 years. What is the nominal annual interest rate if the
interest is compounded monthly? Ans. 8.182%

17. If ₱10,000 is invested at 12% interest compounded annually, determine how many
years it will take to double. Ans. 6.12 years

18. Compare the accumulated amounts after 5 years of ₱1,000 invested at 8% interest
rate per year compounded (a) annually, (b) semiannually, (c) quarterly, (d) monthly,
(e) daily, and (f) continuously. Ans. (a) ₱1,469.33, (b) ₱1,480.24, (c) ₱1,485.95, (d)
₱1489.85, (e) ₱1,491.76, (f) ₱1,491.82

Discrete and Continuous Compounding

19. Find the nominal rate which if compounded quarterly is equivalent to 6.5%
compounded semi-annually. Ans. 6.448%

20. What effective annual interest rate corresponds to a nominal interest rate of 15%
per year, compounded continuously? Ans. 16.18%

21. What nominal interest rate corresponds to an effective interest rate of 12% per
year, compounded continuously? Ans. 11.33%

22. Determine the effective annual interest rate corresponding to a nominal interest
rate of 8.5% per year, if the interest is compounded (a) quarterly, (b) monthly, (c) daily,
(d) continuously. Ans. (a) 8.77%; (b) 8.84%; (c) 8.871%; (d) 8.872%

23. Determine the accumulated value at the end of 10 years if ₱100,000 is invested at
the rate of 6% per year compounded continuously. Ans. ₱182,211.88

24. An investment plan pays 15% per year, compounded continuously. How much
would have to be invested at the end of each year so that ₱40,000 will be accumulated
by the end of 10 years? Ans. ₱1859.26

Concept of Equivalence

25. Suppose that ₱2000 is invested now, ₱2500 two years from now, and ₱1200 four
years from now, all at 8% per year, compounded quarterly. What will be the total
amount 10 years from now? Ans. ₱11,057.33

26. A woman borrowed ₱3,000 to be paid after 1 ½ years with interest at 12%
compounded semiannually and ₱5,000 to be paid after 3 years at 12% compounded
monthly. What single payment must she pay after 3 ½ years at an interest rate of 16%
compounded quarterly to settle the two obligations? Ans. ₱12,627.59

57
Inflation

27. Because of general price inflation in our economy. the purchasing power of the
pesos shrinks with the passage of time. If the inflation rate is expected to be 7% per
year for the foreseeable future, how many years will it take for the pesos' purchasing
power to be one-half of what it is now?

28. Suppose that a person invests ₱30,000 at 10% per year, compounded annually,
for 8 years. (a) Will this effectively protect the purchasing power of the original principal,
given an annual inflation rate of 8%? (b) If so, by how much? Ans. (a) yes; (b) ₱4743.43

Ordinary Annuity

29. A man has deposited ₱50,000 in a retirement income plan with a local bank. This
bank pays 9% per year, compounded annually, on such deposits. What is the
maximum amount the man can withdraw at the end of each year and still have the
funds last for 12 years? Ans. ₱6,982.50

30. Suppose that someone deposits ₱2500 in a savings account at the end of each
year for the next 15 years. How much money will the person have by the end of the
15th year if the bank pays (a) 8%, (b) 6 ¾%, per year, compounded annually? Ans. (a)
₱67,880.28; (b) ₱61,626

31. Mr. De la Cruz has deposited his life savings of ₱700,000 in a retirement income
plan with a local bank. The bank pays 10%, compounded annually, on such deposits.
What is the maximum fixed amount Mr. Jones can withdraw at the end of each year
and still have the funds last for 15 years? Ans. ₱92,031.6

32. How much money would have to be saved at 8% per year, compounded annually,
each year for the next 10 years if ₱50,000 is needed at the end of the 10th year? Ans.
₱3451.47

33. An engineer plans to borrow ₱100,000 to open his own consulting business. He
must repay ₱2,150 a month for 5 years. What is the nominal annual interest rate, based
on monthly compounding? Ans. 10.51%

34. A loan was to be amortized by a group of four end-of-year payments forming an


ascending arithmetic progression. The initial payment was to be ₱5,000 and the
difference between successive payments was to be ₱400. But the loan was
renegotiated to provide for the payment of equal rather than the uniformly varying
sums. If the interest rate of the loan was 15%, what was the annual payment? Ans.
₱5,530.51

35. A new machine is expected to cost ₱60,000 and have a life of 5 years. Maintenance
costs will be ₱15,000 the first year, ₱17,000 the second year, ₱19,000 the third year,
₱21,000 the fourth year, and ₱23,000 the fifth year. To pay for the machine, how much
should be budgeted and deposited in a fund that earns (a) 9% per year, compounded
annually? Ans. ₱132,566.9

36. A person is planning for his retired life. He has 10 more years of service. He would
like to deposit ₱30,000 at the end of the first year and thereafter he wishes to deposit
the same amount (₱30,000) with an annual decrease of ₱2,000 for the next 9 years
with an interest rate of 18%. Find the total amount at the end of the 10th year of the
above series.

58
37. An asphalt road requires no upkeep until the end of 2 years when ₱60,000 is
needed for repairs. After this ₱90,000 will be needed for repairs at the end of each
year for the next 5 years, then ₱120,000 at the end of each year for the next 5 years.
If money is worth 14% compounded annually, what was the equivalent uniform annual
cost for the 12-year period? Ans. ₱79,245

38. Mr. Cruz borrows ₱600,000 at 12% compounded annually, agreeing to repay the
loan in 15 equal payments. How much of the original principal is still unpaid after he
has made the 8th payment? Ans. ₱402,040

Deferred Annuity

39. A person buys a piece of lot for ₱100,000 down payment and 10 deferred semi-
annual payments of ₱8,000, starting 3 years from now. What is the present value of
the investment if the rate of interest is 12% compounded semi-annually? Ans.
₱143,999.08

40. On the day his son was born, a father decided to establish a fund for his son's
college education. The father wants the son to be able to withdraw ₱40,000 from the
fund on his 18th birthday, again on his 19th birthday, again on his 20th birthday, and
again on his 21st birthday. If the fund earns interest at 9% per year, compounded
annually, how much should the father deposit at the end of each year, up through the
17th year? Ans. ₱3504.93

Annuity Due

41. Ms. Reyes deposits ₱7500 in a savings account at the beginning of each year,
starting now, for the next 10 years. If the bank pays interest of 7% per year,
compounded annually, how much money will Ms. Brown have accumulated by the end
of the 10th year? Ans. ₱110,877

42. Determine the present worth and the accumulated amount of an annuity consisting
of 6 payments of ₱120,000 each, the payments are made at the beginning of each
year. Money is worth 15% compounded annually. Ans. ₱52,226.40 ; ₱120,801.60

Perpetuity

43. A fund donated by a wealthy person to IIEE to provide annual scholarships to


deserving EE students. The fund will grant ₱5,000 for each of the first five years,
₱8,000 for the next 5 years, and ₱10,000 for each year thereafter. The scholarship will
start one year after the fund is established. If the fund earns 8% annual interest, what
is the amount of the donation? Ans. ₱99,601.71

Capitalized Cost

44. Calculate the capitalized cost of a project that has an initial cost of ₱3,000,000 and
an additional investment cost of ₱1,000,000 at the end of every ten years. The annual
operating cost will be ₱100,000 at the end of every year for the first four years and

59
₱160,000 thereafter. In addition, there is expected to be recurring major rework cost
of ₱300,000 every 13 years. Assume i = 15%. Ans. ₱4,281,990

45. The will of a wealthy philanthropist left ₱5,000,000 to establish a perpetual


charitable foundation. The foundation trustees decided to spend ₱1,200,000 to provide
facilities immediately and to provide ₱100,000 of capital replacement at the end of
each 5-year period. If the invested funds earned 12% per annum, what would be the
year-end amount available in perpetuity from the endowment for charitable purposes?
₱440,259

Amortization

46. Suppose you borrowed ₱200,000 from a local bank having an interest of 8% per
annum, compounded monthly. If the debt was to be amortized by 36 equal payments
and the first payment is to be made at the end of the first month, (a) determine the
monthly payment, and (b) construct an amortization schedule.

60
CHAPTER 3

DEPRECIATION

One of the concerns that organizations must deal with and account for is that a property
loses its value – even as they continue to function and contribute to the engineering
projects. This loss of value, called depreciation, can involve deterioration and
obsolescence.

On a project level, engineers must be able to assess how the practice of depreciating
properties influences the investment value of a given project. To make this
assessment, they need to estimate the allocation of capital costs over the life of the
project, which requires an understanding of the methods that accountants use. In this
chapter, we will review the methods of asset depreciation.

After completing this chapter, the student shall be able to:

 Define value and valuation;


 Differentiate the types of depreciation; and
 Calculate annual depreciation amounts for tangible assets using straight-line
method, sinking fund method, sum of the years’ digit, and declining balance
method.

Lesson 1: Definitions of Value

Value, in commercial sense, is the present worth of all future profits that are to be
received through ownership of a particular property.

The market value of the property is the amount which a willing buyer will pay to a
willing seller for the property where each has equal advantage and is under no
compulsion to buy or sell.

The utility or use value of a property is what the property is worth to the owner as an
operating unit.

Fair value is the value which is usually determined by a disinterested third party in
order to establish a price that is fair to both seller and buyer.

Book value, sometimes called depreciated book value, is the worth of a property as
shown on the accounting records of an enterprise.

Resale or salvage value is the price that can be obtained from the sale of the property
after it has been used.

Scrap value is the amount the property would sell for if disposed as junk.

Lesson 2: Types of Depreciation

Depreciation is the decrease in the value of physical property with the passage of
time.

61
The different types of depreciation are:

1. Normal depreciation
 Physical depreciation – is due to the lessening of the physical ability of a
property to produce results. Its common causes are wear and tear or
deterioration.
 Functional depreciation – is due to the lessening in the demand for the
function which the property is designed to render. Its common causes are
inadequacy, changes in styles, population centers shift, obsolescence of
technology, saturation of markets, or more efficient machines are produced.

2. Depreciation due to the changes in price levels. This type of depreciation is


almost impossible to predict and therefore is not considered in economic
studies.

3. Depletion. This refers to the decrease in the value of the property due to the
gradual extraction of its contents.

Physical life of a property is the length of time during which it is capable of performing
the function for which it was designed and manufactured.

Economic life is the length of time during which the property may be operated at a
profit.

Lesson 3: Methods of Depreciation

The methods of calculating depreciation are as follows:

1. Straight line method


2. Sinking-fund method
3. Sum of the years’ digits method
4. Declining balance method

These are now discussed in detail.

62
Straight Line Method

This method assumes that the loss in value is directly proportional to the age of the
property (Sta. Maria, 2000).

Co
d

x Dn DT
Co – CL

Cn CL

0 1 2 3 n L

Figure 16. Straight Line Method

Formulas:
𝐷𝑇 𝐶𝑜 − 𝐶𝐿 𝐶𝑜 − 𝐶𝑛
𝑑= = =
𝐿 𝐿 𝑛

𝐶𝑛 = 𝐶𝑜 − 𝐷𝑛 = 𝐶𝑜 − 𝑛𝑑

where:

d – annual depreciation charge, ₱/yr


DT – total depreciation
L – life of the property
Co – first cost
CL – salvage/scrap value
Cn – book value or cost after ‘n’ years
Dn – total depreciation after ‘n’ years
n – nth year

The formula of straight line method is like finding the slope of a line (line x, as shown):

𝑟𝑖𝑠𝑒
𝑚=
𝑟𝑢𝑛
Your slope here is d.

𝐶𝑜 − 𝐶𝐿 𝐶𝑜 − 𝐶𝑛
𝑑= =
𝐿 𝑛

Since, the slope of a straight line is uniform all throughout, you can use similar triangles
in finding book value or depreciation in any time period, to illustrate:

63
DT = Co – CL
Co – C3

3 L

To find for the book value at year 3:

𝐶𝑜 − 𝐶𝐿 𝐶𝑜 − 𝐶3
=
𝐿 3

3(𝐶𝑜 − 𝐶𝐿 )
𝐶3 = 𝐶𝑜 −
𝐿
Examples:

1. A company has purchased an equipment whose first cost is ₱100,000 with an


estimated life of eight years. The estimated salvage value of the equipment at the end
of its lifetime is ₱20,000. (a) Determine the depreciation charge and book value after
5 years using the straight line method of depreciation, and (b) tabulate the
depreciation, total depreciation and book value at the end of various years.

Solution:

Co = ₱100,000 ; CL = ₱20,000 ; L = 8 ; n = 5 ; d = ? ; C5 = ?

(a) To calculate for annual depreciation charge:

𝐶𝑜 − 𝐶𝐿 ₱100,000 − ₱20,000
𝑑= = = ₱10,000
𝐿 8

To calculate for book value after 5 years:

𝐶𝑛 = 𝐶𝑜 − 𝐷𝑛

𝐶𝑛 = 𝐶𝑜 − 𝑛𝑑

𝐶5 = 𝐶𝑜 − 5𝑑

𝐶5 = ₱100,000 − 5(₱10,000) = ₱50,000

Another Solution:
𝐶𝑜 − 𝐶𝐿 𝐶𝑜 − 𝐶5
=
𝐿 5
₱100,000 − ₱20,000 ₱100,000 − 𝐶5
=
8 5

5 (₱80,000)
= ₱100,000 − 𝐶5
8

₱50,000 = ₱100,000 − 𝐶5

𝐶5 = ₱50,000

64
(b)
Table 3. Cn, Dn, and d under Straight Line Depreciation Method

End of Total Depreciation


Depreciation (d) Book Value (Cn)
Year (Dn)
0 ₱ 100,000
1 ₱ 10,000 ₱ 10,000 90,000
2 10,000 20,000 80,000
3 10,000 30,000 70,000
4 10,000 40,000 60,000
5 10,000 50,000 50,000
6 10,000 60,000 40,000
7 10,000 70,000 30,000
8 10,000 80,000 20,000

2. An electronic balance costs ₱90,000 and has an estimated salvage value of ₱8,000
at the end of its 10 years lifetime. What would be the book value after three years,
using the straight line method in solving for the depreciation?

Solution:

Co = ₱90,000 ; CL = ₱8,000 ; L = 10 ; n = 3 ; d = ? ; C3 = ?

𝐶𝑜 − 𝐶𝐿 ₱90,000 − ₱8,000
𝑑= = = ₱8,200
𝐿 10

𝐶3 = 𝐶𝑜 − 𝑛𝑑 = ₱90,000 − 3 (₱8,200) = ₱65,400

Sinking Fund Method

This method assumes that a sinking fund is established in which funds will accumulate
for replacement. The total depreciation that has taken place up to any given time is
assumed to be equal to the accumulated amount in the sinking fund at that time (Sta.
Maria, 2000).

In this method of depreciation, the book value decreases at increasing rates with
respect to the life of the asset (Panneerselvam, 2001).

To calculate for the total depreciation:

(1 + 𝑖)𝐿 − 1
𝐷𝑇 = 𝐶𝑜 − 𝐶𝐿 = 𝑑 [ ]
𝑖

From the formula above, the annual depreciation is

(𝐶𝑜 − 𝐶𝐿 ) 𝑖
𝑑=
(1 + 𝑖)𝐿 − 1

65
Co
Dn DT

Cn CL

0 1 2 3 ... n L

d d d d d d d

Figure 17. Sinking Fund Method

Total depreciation at “n” years:

(1 + 𝑖)𝑛 − 1
𝐷𝑛 = 𝑑 [ ]
𝑖
Book value at “n” years:

𝐶𝑛 = 𝐶𝑜 − 𝐷𝑛

where:
d – annual depreciation cost
DT – total depreciation
L – life of the property
Co – first cost
CL – salvage/scrap value
Cn – book value or cost after ‘n’ years
Dn – total depreciation after ‘n’ years
n – nth year
i – interest

This method is similar to finding the uniform payment series sinking fund, with A being
d and F being DT or CO – CL . This topic is under Ordinary Annuity.

𝑖
𝐴 = 𝐹[ ]
(1 + 𝑖)𝑛 − 1

Examples:

1. A company has purchased an equipment whose first cost is ₱100,000 with an


estimated life of eight years. The estimated salvage value of the equipment at the end
of its lifetime is ₱20,000. (a) Determine the depreciation charge and book value after
5 years using the straight line method of depreciation, and (b) tabulate the
depreciation, total depreciation and book value at the end of various years. Assume
interest rate of 12%, compounded annually.

66
Solution:

Co = ₱100,000 ; CL = ₱20,000 ; L = 8 ; n = 5 ; i = 12% ; d = ? ; C5 = ?

To calculate for annual depreciation cost (this is fixed annually):

(𝐶𝑜 − 𝐶𝐿 ) 𝑖 (₱100,000 − ₱20,000) (0.12)


𝑑= 𝐿 = = ₱6504.23
(1 + 𝑖) − 1 (1 + 0.12)8 − 1

To calculate for book value after 5 years:

𝐶𝑛 = 𝐶𝑜 − 𝐷𝑛

(1 + 𝑖)𝑛 − 1
𝐶𝑛 = 𝐶𝑜 − 𝑑 [ ]
𝑖

(1 + 0.12)5 − 1
𝐶5 = ₱100,000 − ₱6504.23 [ ]
0.12

𝐶5 = ₱58,679.62

Table 4. Cn, Dn, and d under Sinking Fund Depreciation Method

End of Total Depreciation


Fixed Depreciation (d) Book Value (Cn)
Year (Dn)
0 ₱ 100,000
1 ₱ 6,504.23 ₱ 6,504.23 93,495.77
2 6,504.23 13,788.97 86,211.03
3 6,504.23 21,947.87 78,052.13
4 6,504.23 31,085.85 68,914.15
5 6,504.23 41,320.38 58,679.62
6 6,504.23 52,783.06 47,216.94
7 6,504.23 65,621.25 34,378.75
8 6,504.23 80,000.00 20,000.00

2. A firm bought an equipment for ₱56,000. Other expenses including installation


amounted to ₱4,000. The equipment is expected to have a life of 16 years with a
salvage value of 10% of the original cost. Determine the book value at the end of 12
years by sinking fund method at 12% interest.

Solution:

Co = ₱56,000 + ₱4,000 = ₱60,000 ; CL = ₱60,000 (0.10) = ₱6,000 ; L = 16 ; n = 12 ; i


= 12% ; C12 = ?

(₱60,000 − ₱6,000) (0.12)


𝑑= = ₱1,263
(1 + 0.12)16 − 1

(1 + 𝑖)𝑛 − 1 (1 + 0.12)12 − 1
𝐶12 = 𝐶𝑜 − 𝐷12 = 𝐶𝑜 − 𝑑 [ ] = ₱60,000 − ₱1,263 [ ]
𝑖 0.12
= ₱29,520

67
Sum of the Years’ Digits (SYD) Method

In this method, it is assumed that the book value of the property decreases at a
decreasing rate.

The rate of depreciation charge for the first year is assumed as the highest and then it
decreases. For any year, the depreciation is calculated by multiplying the
corresponding rate of depreciation with (C O – CL).

Depreciation at “n” year:

𝑑𝑛 = (𝑟𝑎𝑡𝑒 𝑜𝑓 𝑑𝑒𝑝𝑟𝑒𝑐𝑖𝑎𝑡𝑖𝑜𝑛)(𝑡𝑜𝑡𝑎𝑙 𝑑𝑒𝑝𝑟𝑒𝑐𝑖𝑎𝑡𝑖𝑜𝑛)

𝑟𝑒𝑣𝑒𝑟𝑠𝑒 𝑑𝑖𝑔𝑖𝑡
𝑑𝑛 = (𝐶 − 𝐶𝐿 )
𝑠𝑢𝑚 𝑜𝑓 𝑑𝑖𝑔𝑖𝑡𝑠 𝑜
Total depreciation:

𝐷𝑇 = 𝑑1 + 𝑑2 + 𝑑3 + ⋯ + 𝑑𝐿 = 𝐶𝑜 − 𝐶𝐿

For example, for a property whose life is 5 years. The sum of the years’ digits is
computed as

𝑆 = 1 + 2 + 3 + 4 + 5 = 15
or by sum of arithmetic series
𝑛 5
𝑆 = (1 + 𝑛) = (1 + 5) = 15
2 2

The rates of depreciation for the years 1–5, respectively are as follows: 5/15, 4/15,
3/15, 2/15, and 1/15.

Year in
Rate of Depreciation during
Year Reverse
Depreciation the Year
Order
1 5 5/15 5/15 (Co – CL)
2 4 4/15 4/15 (Co – CL)
3 3 3/15 3/15 (Co – CL)
4 2 2/15 2/15 (Co – CL)
5 1 1/15 1/15 (Co – CL)
Σ of Digits = 15

Thus,
𝑑1 ≠ 𝑑2 ≠ 𝑑3 ≠ ⋯ ≠ 𝑑𝑛

Examples:

1. A company has purchased an equipment whose first cost is ₱100,000 with an


estimated life of eight years. The estimated salvage value of the equipment at the end
of its lifetime is ₱20,000. (a) Determine the depreciation charge and book value after
5 years using the straight line method of depreciation, and (b) tabulate the
depreciation, total depreciation and book value at the end of various years.

Solution:

Co = ₱100,000 ; CL = ₱20,000 ; L = 8 ; n = 5 ; d = ? ; C5 = ?

68
To calculate for annual depreciation cost:

𝑛 8
𝑆= (1 + 𝑛) = (1 + 8) = 36
2 2
𝑟𝑒𝑣𝑒𝑟𝑠𝑒 𝑑𝑖𝑔𝑖𝑡
𝑑𝑛 = (𝐶 − 𝐶𝐿 )
𝑠𝑢𝑚 𝑜𝑓 𝑑𝑖𝑔𝑖𝑡𝑠 𝑜

8
𝑑1 = (₱100,000 − ₱20,000) = ₱17,777.77
36
7
𝑑2 = (₱80,000) = ₱15,555.55
36
6
𝑑3 = (₱80,000) = ₱13,333.33
36

5
𝑑4 = (₱80,000) = ₱11,111.11
36
4
𝑑5 = (₱80,000) = ₱8,888.88
36
3
𝑑6 = (₱80,000) = ₱6,666.66
36
2
𝑑7 = (₱80,000) = ₱4,444.44
36
1
𝑑8 = (₱80,000) = ₱2,222.22
36

To calculate for book value after 5 years:

𝐿−𝑛 𝐿−𝑛+1
𝐶𝑛 = (𝐶𝑜 − 𝐶𝐿 ) ∙ ∙ + 𝐶𝐿
𝐿 𝐿+1

8−5 8−5+1
𝐶𝑛 = (₱100,000 − ₱20,000) ∙ ∙ + ₱20,000
8 8+1

𝐶𝑛 = ₱33,333.33

Table 5. Cn, Dn, and d under Sum of the Years’ Digits Depreciation Method

End of Total Depreciation


Depreciation (d) Book Value (Cn)
Year (Dn)
0 ₱ 100,000
1 ₱ 17,777.78 ₱ 17,777.78 82,222.22
2 15,555.56 33,333.33 66,666.67
3 13,333.33 46,666.67 53,333.33
4 11,111.11 57,777.78 42,222.22
5 8,888.89 66,666.67 33,333.33

69
6 6,666.67 73,333.33 26,666.67
7 4,444.44 77,777.78 22,222.22
8 2,222.22 80,000.00 20,000.00

2. A consortium of international telecommunication companies contracted for the


purchase and installation of a fiber optic cable linking two major cities at a total cost of
US$ 960 million. This amount includes freight and installation charges estimated at
10% of the above contract price. If the cable shall be depreciated over a period of 15
years with a zero salvage value, what is the depreciation charge during the 8 th year
using the sum-of-years-digits method?

Solution:

Co = $960,000,000 ; CL = 0 ; L = 15 ; d = ? ; C5 = ?

To get the sum of the digits:


𝑛 15
𝑆= (1 + 𝑛) = (1 + 15) = 120
2 2

𝑅𝑒𝑣𝑒𝑟𝑠𝑒 𝑑𝑖𝑔𝑖𝑡 𝑐𝑜𝑟𝑟𝑒𝑠𝑝𝑜𝑛𝑑𝑖𝑛𝑔 𝑡𝑜 𝑡ℎ𝑒 8𝑡ℎ 𝑦𝑒𝑎𝑟 𝑜𝑓 𝑙𝑖𝑓𝑒 = 8

8
𝑑8 = ($960,000,000) = $64,000,000
120

Declining Balance Method

In this method, sometimes called Constant Balance Method, Fixed Percentage


Method, or Matheson’s Formula, it is assumed that the annual cost of depreciation is
a fixed percentage of the salvage value at the beginning of the year. The ratio of the
depreciation in any year to the book value at the beginning of that year is constant
throughout the life of the property and is designated by k, the rate of depreciation.

𝐿 𝐶𝐿 𝑛 𝐶𝑛
𝑘 =1− √ =1− √
𝐶𝑜 𝐶𝑜

The depreciation during the year is given by

𝑑𝑛 = 𝐶𝑜 (1 − 𝑘)𝑛−1 𝑘

To calculate for the book value after “n” year:

𝐶𝑛 = 𝐶𝑜 (1 − 𝑘)𝑛
To calculate for the salvage value:

𝐶𝐿 = 𝐶𝑜 (1 − 𝑘)𝐿

The book value at the end of the life of the property may not be exactly equal to the
salvage value of the property. This is a major limitation of this approach.

where:

70
k – annual rate of depreciation
dn – annual depreciation
Co – first cost
CL – salvage/scrap value
Cn – book value or cost after ‘n’ years
L – life of the property
n – nth year

Examples:

1. A company has purchased an equipment whose first cost is ₱100,000 with an


estimated life of eight years. The estimated salvage value of the equipment at the end
of its lifetime is ₱20,000. (a) Determine the depreciation charge and book value after
5 years using the straight line method of depreciation, and (b) tabulate the
depreciation, total depreciation and book value at the end of various years.

Solution:

Co = ₱100,000 ; CL = ₱20,000 ; L = 8 ; n = 5 ; d = ? ; C5 = ?

To calculate for the annual rate of depreciation:

𝐿 𝐶𝐿 8 ₱20,000
𝑘 =1− √ = 1− √ = 0.1822 = 18.22%
𝐶𝑜 ₱100,000

To find for depreciation:

𝑑𝑛 = 𝐶𝑜 (1 − 𝑘)𝑛−1 𝑘

𝑑1 = ₱100,000 (1 − 0.1822)1−1 (0.1822) = ₱18,220.00

𝑑2 = ₱100,000 (1 − 0.1822)2−1 (0.1822) = ₱14,900.32

𝑑3 = ₱100,000 (1 − 0.1822)3−1 (0.1822) = ₱12,185.48

𝑑4 = ₱100,000 (1 − 0.1822)4−1 (0.1822) = ₱9,965.28

𝑑5 = ₱100,000 (1 − 0.1822)5−1 (0.1822) = ₱8,149.61

𝑑6 = ₱100,000 (1 − 0.1822)6−1 (0.1822) = ₱6,664.75


𝑑7 = ₱100,000 (1 − 0.1822)7−1 (0.1822) = ₱5,450.43

𝑑8 = ₱100,000 (1 − 0.1822)8−1 (0.1822) = ₱4,457.36

To calculate for the book value at year 5:

𝐶𝑛 = 𝐶𝑜 (1 − 𝑘)𝑛

𝐶5 = ₱100,000 (1 − 0.1822)5

𝐶5 = ₱36,579.31

71
Table 6. Cn, Dn, and d under Declining Balance Method Depreciation Method

End of Total Depreciation


Depreciation (d) Book Value (Cn)
Year (Dn)
0 ₱ 100,000.00
1 ₱ 18,820.00 ₱ 18,220.00 81,780.00
2 14,900.32 33,120.32 66,879.68
3 12,185.48 45,305.79 54,694.21
4 9,965.28 55,271.08 44,728.92
5 8,149.61 63,420.69 36,579.31
6 6,664.75 70,085.44 29,914.56
7 5,450.43 75,535.87 24,464.13
8 4,457.36 79,993.24 20,006.76

2. Determine the rate of depreciation, the total depreciation up to the end of 8th year
and the book value at the end of 8 years for an asset that costs ₱15,000 new and has
an estimated scrap value of ₱2,000 at the end of 10 years by the declining balance
method.

Solution:

Co = ₱15,000 ; CL = ₱2,000 ; L = 10 ; n = 8 ; k = ? ; C8 = ? ; D8 = ?

𝐿 𝐶𝐿 10 ₱2,000
𝑘 =1− √ =1− √ = 0.1825 = 18.25%
𝐶𝑜 ₱15,000

𝐶8 = 𝐶𝑜 (1 − 𝑘)𝑛 = ₱15,000 (1 − 0.1825)8 = ₱2,992

𝐷8 = 𝐶𝑜 − 𝐶8 = ₱15,000 − ₱2,992 = ₱12,483

Lesson 4: Valuation

Valuation or appraisal is the process of determining the value of certain property for
specific reasons. The person engaged in the task of valuation is called an appraiser.

Intangible values:

In the determination of the value of industrial property or equipment, four intangible


items are often encountered.

 Goodwill is that element of value which a business has earned through the
favorable consideration and patronage of its customers arising from its well-
known and well conducted policies and operation.
 Franchise is an intangible item of value arising from the exclusive right of a
company to provide a specific product or service in a stated region of the
country.
 Going value is an intangible value which an actually operating concern has due
to its operation.
 Organization cost is the amount of money spent in organizing a business and
arranging for its financing and building.

72
CHAPTER EXERCISES

Objective Test 3

1. It is defined as the reduction or fall of the value of an asset due to constant use and
passage of time?

a. depletion b. inflation c. depreciation d. deflation

2. In what method of computing depreciation where it assumes that the loss in value
is directly proportional to the age of the equipment or asset?

a. straight line method b. sinking fund method


c. sum of the year’s digit method d. declining balance method

3. In what method of computing depreciation where it assumes that a sinking fund is


established in which funds will accumulate for replacement purposes?

a. straight line method b. sinking fund method


c. sum of the year’s digit method d. declining balance method

4. In what method of computing depreciation where it assumes that the annual cost of
depreciation is a fixed percentage of the book value at the beginning of the year?

a. straight line method b. sinking fund method


c. sum of the year’s digit method d. declining balance method

5. In SYD method of computing depreciation, which of the following is the formula in


finding the sum of the years’ digits?

a. n (n – 1) b. n (n + 2) / 2 c. n (n – 1) / 2 d. n (n + 1) / 2

6. The declining balance method is also known as _____________.

a. double percentage method b. constant percentage method


c. modified sinking fund method d. modified SYD method

7. What type of depreciation is due to the reduction in the demand for the function that
the equipment or asset was designed to render?

a. functional depreciation b. design depreciation


c. physical depreciation d. demand depreciation

8. What type of depreciation is due to the reduction of the physical ability of an


equipment or asset to produce results?

a. functional depreciation b. design depreciation


c. physical depreciation d. demand depreciation

9. What is defined as the reduction of the value of a certain natural resources such as
mines, oil, timber, quarries, etc. due to the gradual extraction of its contents?

a. depletion b. inflation c. depreciation d. deflation

10. The amount of property in which a willing buyer will pay to a willing seller for the
property when neither one is under the compulsion to buy nor sell is called ________.

a. fair value b. market value c. goodwill value d. book value

73
11. Salvage value is sometimes known as ____________.

a. scrap value b. second-hand value c. junk value d. use value

12. What refers to the value of an asset which a disinterested third party, different form
the buyer and seller, will determine in order to establish a price acceptable to both
parties?

a. book value b. market value c. fair value d. franchise value

13. The process of determining the value or worth of a physical property for specific
reason is called _____________.

a. investment b. valuation c. economy d. depletion

14. The unrecovered depreciation which results due to poor estimates as to the life of
the equipment is called ____________.

a. sunk cost b. economic life c. in-place value d. annuity

15. Capitalized cost of any structure or property is computed by which formula?

a. first cost + interest of first cost b. annual cost + interest of the first cost
c. first cost + cost of perpetual maintenance d. first cost + salvage value

16. What refers to the value of an intangible item which arises from the exclusive right
of a company to provide a specified product and service in a certain region of the
country?

a. company value b. going value c. goodwill value d. franchise value

17. ___________ is an element of value which a business has earned through the
favorable consideration and patronage of its customers arising from its well-known and
well conducted policies and operations.

a. status of company b. big income c. known owners d. goodwill

18. The person engaged in the task of valuation is called

a. evaluator b. appraiser c. estimator d. broker

Problem Solving 3

Straight line Method:

1. A tax and duty free importation of a 30 HP sandmill (for paint manufacturing) cost
₱360,000, CIF Manila. Bank charges, arrastre and brokerage cost ₱5,000. Foundation
and installation costs were ₱25,000. Other incidental expenses amounted to ₱20,000.
Salvage value of the mill is estimated to be ₱60,000 after 20 years. Find the appraisal
value of the mill, using straight-line depreciation, at the end of (a) 10 years and (b) 15
years. Ans. (a) ₱235,000 (b) ₱147,500

74
2. A telephone company purchased a microwave radio equipment for ₱6,000,000.
Freight and installation charges amounted to 3% of the purchase price. If the
equipment shall be depreciated over a period of 8 years with a salvage value of 5%,
determine the annual depreciation charge using the straight-line method. Ans.
₱733,875

3. A machine shop purchased 10 years ago a milling machine for ₱60,000. A straight-
line depreciation reserve had been provided on a 20-year life of the machine. The
owner of the machine shop desires to replace the old milling machine with a modern
unit of many advantages costing ₱100,000. It can sell the old unit for ₱20,000. How
much new capital will be required for the purchase? Ans. ₱50,000

Sinking Fund Method:

4. Power to a remote transmitting station is provided by a Diesel-electric generator


unit. The original cost of the unit is ₱65,000. It costs ₱2,000 to ship the unit to the job
site. An additional cost of ₱3,000 was incurred for installation. Determine the annual
depreciation cost by the sinking fund method, if the unit has an expected life of 10
years. The salvage value of the unit at the end of its life was estimated at ₱5,000.
Assume that the annual charge for depreciation was deposited in a fund drawing
compound interest at the rate of 5%. Ans. ₱5,168

5. An industrial plant bought a generator set for ₱90,000. Other expenses including
installation amounted to ₱10,000. The generator set is to have a life of 17 years with
a salvage value at the end of life of ₱5,000. Determine the depreciation charge during
the 13th year and the book value at the end of 13 years by the sinking fund method at
12%. Ans. ₱1,943; ₱45,539

SYD Method:

6. An industrial plant bought a generator set for ₱90,000. Other expenses including
installation amounted to ₱10,000. The generator set is to have a life of 17 years with
a salvage value at the end of life of ₱5,000. Determine the depreciation charge during
the 13th year and the book value at the end of 13 years by SYD method. Ans. (a)
₱3,105; ₱11,209

7. A telephone company purchased a microwave radio equipment for ₱6,000,000.


Freight and installation charges amounted to 3% of the purchase price. If the
equipment shall be depreciated over a period of 8 years with a salvage value of 5%,
determine the depreciation charge during the fifth year using the sum-of-the-years-digit
method. Ans. ₱652,333

Declining Balance Method:

8. An industrial plant bought a generator set for ₱90,000. Other expenses including
installation amounted to ₱10,000. The generator set is to have a life of 17 years with
a salvage value at the end of life of ₱5,000. Determine the depreciation charge during
the 13th year and the book value at the end of 13 years by declining balance method.
Ans. ₱1,949, ₱10,118

75
CHAPTER 4

CAPITAL FINANCING

To obtain money or funds in order to get the business off the ground or help in the daily
operations of the business such as the purchase of materials and payment of wages
etc., an individual or organization usually raise funds through capital financing
programs. In this chapter, we are going to study the different forms of business
organization and the methods to set up funds for these businesses.

After completing this chapter, the student shall be able to:

 Define equity and borrowed funds;


 Differentiate the types of business organization; and
 Define stocks and bonds.

Lesson 1: Types of Business Organizations

A. Individual Ownership. The individual ownership or sole proprietorship is the


simplest form of business organization, wherein a person uses his or her own capital
to establish a business and is the sole owner.

Advantages:

1. It is easy to organize.
2. The owner has full control of the enterprise.
3. The owner is entitled to whatever benefits and profits that accrue from the
business.
4. It is easy to dissolve.
Disadvantages:

1. The amount of equity capital which can be accumulated is limited.


2. The organization ceases upon the death of the owner.
3. It is difficult to obtain borrowed capital, owing to the uncertainty of the life of the
organization.
4. The liability of the owner for his debts is unlimited.

B. The Partnership. A partnership is an association of two or more persons for the


purpose of engaging in a business for profit.

Advantages:

1. More capital may be obtained by the partners pooling their resources together.
2. It is bound by few legal requirements as to its accounts, procedures, tax
reforms and other items of operation.
3. Dissolution of the partnership may take place at any time by mere agreement
of the partners.

76
4. It provides an easy method whereby two or more persons of differing talents
may enter into business, each carrying those burdens that he can best handle.
Disadvantages:

1. The amount of capital that can be accumulated is definitely limited.


2. The life of the partnership is determined by the life of the individual partners.
When any partner dies, the partnership automatically ends.
3. There may be serious disagreement among the individual partners.
4. Each partner is liable for the debts of the partnership.
C. The Corporation. A corporation is a distinct legal entity, separate from the
individuals who own it, and which can engage in almost any type of business
transaction in which a real person could occupy himself or herself.

Advantages:

1. It enjoys perpetual life without regard to any change in the person of its owners,
the stockholders.
2. The stockholders of the corporation are not liable for the debts of the
corporation.
3. It is relatively easier to obtain large amounts of money for expansion, due to its
perpetual life.
4. The ownership in the corporation is readily transferred.
5. Authority is easily delegated by the hiring of managers.
Disadvantages:

1. The activities of a corporation are limited to those stated in its charter.


2. It is relatively complicated in formation and administration.
3. There is a greater degree of governmental control as compared to other types
of business organizations.

Lesson 2: Equity and Borrowed Capital

Other than using up one’s savings, there are usually two types of capital used by
companies to fund their business activities:

 Equity capital or ownership funds are those supplied and used by the owners
of an enterprise in the expectation that a profit will be earned. Debt capital is
usually raised by selling shares of stock.

 Borrowed capital or debt funds are those supplied by others on which a fixed
rate of interest must be paid and the debt must be repaid at a specified time.
Borrowed capital is usually raised by obtaining bank loans and bonds.

Lesson 3: Stocks

Stocks are equity investment that represents part ownership in a corporation and
entitles you to part of the corporation’s earnings and assets.

77
 A share of stock is the single smallest denomination of a company’s stock.
 A dividend is a sum of money paid regularly by a company to its shareholders
out of its profits.
The capital of a corporation is acquired through the sale of stock. There are two
principal types of capital stock: common stock and preferred stock.

Common Stock:

Common stock represents ordinary ownership without special guarantees of return.


Common stockholders have certain legal rights, among which are the following:

1. Vote of stockholders’ meetings.


2. Elect directors and delegates to them power to conduct the affairs of the
business.
3. Sell or dissolve the corporation.
4. Make and amend the by-laws of the corporation.
5. Subject to government approval, amend, or change the charter or capital
structure.
6. Participate in the profits.
7. Inspect the books of the corporation.

Preferred Stock:

Preferred stockholders are guaranteed a definite dividend on their stocks. In case the
corporation is dissolved, the assets must be used to satisfy the claims of the preferred
stockholders before those of the holders of the common stock. Preferred stockholders
usually have the right to vote in meetings, but not always.

Lesson 4: Bonds

A bond is a certificate of indebtedness of a corporation usually for a period not less


than ten years and guaranteed by a mortgage on certain assets of the corporation or
its subsidiaries. Bonds are issued when there is a need for more capital such as for
expansion of the plant or the services rendered by the corporation.

The face or par value of a bond is the amount stated on the bond. When the face
value has been repaid, the bond is said to have been retired or redeemed. The bond
rate is the interest rate quoted on the bond.

Classification of Bonds:

1. Registered bonds. The name of the owner of this bond is recorded on the record
books of the corporation and interest payments are sent to the owner periodically
without any action on his part.

2. Coupon bonds. Coupon bond have coupon attached to the bond for each interest
payment that will come due during the life of the bond. The owner of the bond can
collect the interest due by surrendering the coupon to the offices of the corporation or
at specified banks.

78
CHAPTER EXERCISES

Objective Test 4

1. A form a business organization in which a person conducts his business alone and
entirely for his own profit, being solely responsible for all its activities and liabilities.
a. sole proprietorship b. entrepreneurship c. partnership d. corporation

2. Is an artificial created by operation of law, having the right of succession and the
process, attributes, and properties expressly authorized by the law or incident to its
existence.
a. corporation b. property c. partnership d. organization

3. In case of bankruptcy of a partnership,


a. the partners are not liable for the liabilities of the partnership
b. the partnership assets (excluding the partner’s personal assets) only will be used to
pay the liabilities
c. the partner’s personal assets are attached to the debt of the partnership
d. the partners may sell stock to generate additional capital

4. Which is true about corporation?


a. It is the worst type of business organization.
b. The minimum number of incorporators to start a corporation is three.
c. Its life is dependent on the lives of the incorporators.
d. The stockholders of the corporation are only liable to the extent of their investments.

5. An association of two or more persons for the purpose of engaging into a business
for profit is called
a. entrepreneurship b. partnership c. proprietorship d. corporation

6. What represents the ownership of stockholders who have a residual claim on the
assets of the corporation after all other claims have been settled?
a. authorized capital stock b. preferred stock
c. incorporator’s stock d. common stock

7. The amount of company’s profit that the board of directors of the corporation decides
to distribute to ordinary shareholders is called __________.
a. dividend b. return c. share of stock d. equity

8. _________ is a loan that a company puts out to raise capital.


a. fund b. bond c. stock d. share

9. Cash money and credit necessary to establish and operate an enterprise are
generally called ___________.
a. capital b. funds c. assets d. liabilities

10. What represents the share of participation in business organizations?


a. franchise b. partnership c. stock d. corporation

79
REFERENCES

Textbooks:

Excel Academic Council. Engineering Sciences and Allied Subjects. First Benchmark
Publisher, Inc. (2008)

Panneerselvam, R. Engineering Economics. PHI Learning Private Limited (2012)

Park, Chan S. Fundamentals of Engineering Economics. Pearson Education, Inc.


(2004)

Sepulveda, Jose A. et al. Schaum’s Outline of Theory and Problems of Engineering


Economics. The McGraw-Hill Companies. (1984)

Sta. Maria, Hipolito B. Engineering Economy, 3rd edition. National Bookstore (2000)

Internet Sources:

https://www.marketing91.com/difference-between-goods-and-services/

https://www.quora.com/What-is-the-difference-between-elastic-and-inelastic-
demand/answer/Chanchal-Gupta-12

https://www.quora.com/What-is-the-law-of-diminishing-returns/answer/Shubho-
Dasgupta

http://facstaff.cbu.edu/~gmcginni/classes/CE%20314%20Engineering%20Economy/
PowerPoint/Chapter%201%20pp%201-%2023.pdf

https://www.cleverism.com/skills-and-tools/capital-raising-skills/

80

You might also like